Download as pdf or txt
Download as pdf or txt
You are on page 1of 168

PEV 107

VERBAL ABILITY II

1
Table of Contents

S. No. Chapter Page No.

1 Sentence Correction 3-24

Voice & Accent 25-34


2

Stress & Intonation 35-60

Vocabulary 61-82
3

Email writing 83-88

Essay Writing 89-109


4

Reading Comprehension 110-129

Direct-Indirect Speech 130-141


5

Cover Letter 142-152

6 Critical Reasoning 153-169

2
UNIT 1
SENTENCE CORRECTION
Introduction

◻ Words, phrases and sentences are the bases of any language.

◻ Knowing the common structures and the nuances of the language is crucial

◻ Sentence correction is a type of question

◻ Two variants - spotting error and improving the sentence

◻ Involve errors based on grammar concepts of the language being tested

Eight Types of Errors in Sentence Correction

◻ Subject-Verb Agreement

◻ Pronoun Agreement

◻ Modifiers

◻ Parallelism

◻ Comparisons

◻ Redundancy

◻ Error of Participles

◻ Verb Tenses

Subject - Verb Agreement

In the present tense, a verb must agree in number with its subject. That's the basic principle of subject-verb
agreement. It's a simple enough rule, but on certain occasions even experienced writers can slip up on it.

Let's have a look at three of the trickier cases of subject-verb agreement:

CASE #1: Making Subject and Verb Agree When Words Come Between Them

3
In determining subject-verb agreement, don't let yourself be confused by words that come between the
subject and the verb.

● The country having the most lakes is Canada. [The true subject is country, not lakes. Therefore, a
singular verb is needed.]

● The dog that has all the cuts was in a fight. [The true subject is dog, not cuts. Therefore, a singular
verb is needed.]

● The little girl, along with all her friends, plays the violin. [The true subject is girl, not friends.
Therefore, a singular verb is needed.]

● Skating, in addition to hockey and curling, is offered at the local arena. [The true subject is skating,
not hockey and curling. Therefore, a singular verb is needed.]

● The manager, together with his employees, works late on Thursdays. [The true subject is manager,
not employees. Therefore, a singular verb is needed.]

CASE #2: Reaching Agreement When the Subject Is an Indefinite Pronoun

Remember to add an -s to the end of the verb in the present tense if the subject is one of the indefinite
pronouns listed below:

● anyone (everyone, no one, someone)

● anybody (everybody, somebody, nobody)

● anything (everything, something, nothing)

● each, either, neither

In the following sentences, each subject is an indefinite pronoun and each verb ends in -s:

● Nobody claims to be perfect.

● Everybody plays the fool sometimes.

● Each of the divers has an oxygen tank.

In that last sentence, note that ―has‖ agrees with the subject ―each‖, not with ―divers‖ (the object of the
preposition).

CASE #3: Making Have, Do, and Be Agree with Their Subjects

Many agreement errors result from the misuse of the common verbs ―have, do, and be.‖

We need to remember that the verb ―have‖ appears as ―has‖ if the subject is a singular noun or a third-
person singular pronoun (he, she, it):

● Dana Barrett has ghosts in her bedroom.


4
If the subject is a plural noun or the pronoun ―I, you, we, or they‖, use ―have‖:

● The Ghostbusters have a new client.

In a nutshell, "She has," but "They have."

Similarly, the verb ―do‖ appears as ―does‖ if the subject is a singular noun or, once again, a third-person
singular pronoun (he, she, it):

● Gus does the housework.

If the subject is a plural noun or the pronoun ―I, you, we, or they‖, use ―do‖:

● Gus and Martha do the chores together.

The verb ―be‖ has three forms in the present tense: is, am, are. Use ―is‖ if the subject is a singular noun or a
third-person singular pronoun (he, she, it):

● Dr.Venkman is unhappy.

Use ―am‖ if the subject is the first-person singular pronoun (I):

● I am not the person you think I am.

Finally, if the subject is a plural noun or the pronoun ―you, we, or they‖, use ―are‖:

● The fans are in the stands, and we are ready to play.

Now, let's take one more look at these three verbs—-but from a different angle.

Sometimes a subject may follow (rather than proceed) forms of the verb ―have, do, and be‖. As shown in the
sentences below, this reversal of the usual order occurs in questions that require a helping verb:

● Where has Egon parked the car?

● What do you do in your free time?

● Are we having a test today?

In all of these sentences, the present forms of ―have, do, and be‖ serve as helping verbs and appear in front
of their subjects. Another case in which a form of the verb comes before the subject is in sentences
beginning with the words ―there or here‖:

● There is a unicorn in the garden.

● Here are the photocopies.

5
Pronoun antecedent Error

Correct pronouns match their antecedent (the word they stand in for). For example, in the sentence, ―Sara
took off her hat,‖ ―Sara‖ is the antecedent for ―her.‖ Plural pronouns (we, they, them, us) match with
plural antecedents, and singular pronouns (I, he, she, her, him, it) match with singular antecedents.

Let‘s look at one example: I never go to that restaurant because they have moldy cheese.

Who has moldy cheese? We can‘t tell from the sentence. ―They‖ is a plural pronoun, and its possible
antecedents (―I‖ and ―restaurant‖) are both singular. We would rewrite it like this:

I never go to that restaurant because it has moldy cheese.

Let‘s Practice 2: While the Senate of the Roman Republic did not have the power to enact laws, their
decrees generally were obeyed like law, and through them they exercised considerable influence.

(A) their decrees generally were obeyed like law, and through them they

(B) its decrees generally were obeyed as if to be law, and through these it

(C) their decrees, generally obeyed as law, through it they

(D) its decrees generally were obeyed like law, and through them it

(E) their decrees were generally obeyed as was the law, and through it they

Modifiers

Consider the following example: Looking out the window, the trees were seen by her.

Look at the part of the sentence before the comma. This is the modifier. It modifies or changes the rest of the
sentence in some way.

● What is the verb in the modifier? The verb is ―looking out‖.

● What is the subject of the verb? You may notice that in the modifier there is no subject for this verb.
This makes the modifier a dangling modifier. It is not connected to the rest of the sentence properly. The
rule is that the correct subject of the verb must come immediately after the comma.

● To determine what the subject is, you can ask yourself the question: Who or what performed the action?
The answer to that question gives you the subject. The answer is that ―she‖ did the looking. It cannot be
the trees looking out the window. Therefore, ―she‖ must come directly after the comma.

The correctly written sentence would become: Looking out the window, she saw the trees.

In a dangling modifier, the modifying phrase is misplaced, describing the incorrect word or phrase so
that the sentence is illogical. Look at the following sentence:

A beautiful red-haired mermaid, the president thought Ariel was highly intelligent.
6
This sentence contains a dangling modifier: The president is likely not a beautiful red-haired mermaid
(though that would be cool). The modifier ―A beautiful red-haired mermaid‖ belongs next to the noun it
modifies, which is Ariel. Let‘s rewrite it to remove the dangling modifier:

The president thought Ariel, a beautiful red-haired mermaid, was highly intelligent.

Let‘s Practice 3: Between 1892 and 1893, Claude Monet produced a series of paintings of the Rouen
Cathedral, revised in his studio in 1894, and with the French public receiving it as an emblem of all that
was noble about their history and customs.

(A) produced a series of paintings of the Rouen Cathedral, revised in his studio in 1894, and with the
French public receiving it

(B) produced a series of paintings of the Rouen Cathedral, which he revised in his studio in 1894 and which
the French public received

(C) produced a series of paintings of the Rouen Cathedral, which he revised in his studio in 1894, and that
the French public received it

(D) painted the Rouen Cathedral, which he revised in his studio in 1894, and that the French public
received it

(E) painted the Rouen Cathedral, revised in his studio in 1894, and the French public received it

Parallelism

While forming a sentence, the structure of the sentence should be kept parallel. If an infinitive is used, then
all the phrases should have an infinitive.

Example: She likes to cook, dance and play.

Similar rule is used for a gerund.

Example: She likes cooking, dancing and playing.

Let‘s Practice 4: With companies spending large parts of their advertising budgets online, the market for
content such as feature articles and opinion essays created by a professional writer, a blogger, and by
individual users, are expanding rapidly.

A. the market for content such as feature articles and opinion essays created by a professional writer, a
blogger, and by individual users, are expanding

B. the market for content such as feature articles and opinion essays created by professional writers,
bloggers, and by individual users, are expanding

C. the market for content such as feature articles and opinion essays created by a professional writer, a
blogger, and individual users, is expanding

7
D. the market for content such as feature articles and opinion essays created by professional writers,
bloggers, and individual users, are expanding

8
E. the market for content such as feature articles and opinion essays created by professional writers,
bloggers, and individual users, is expanding

Comparisons

The comparisons made should be between two similar things. If we say: - The population of London is
greater than any other city in India.

We are comparing: - (a) The population of London (b) Any other city in India.

While comparison had to be made between the populations of both. So, the correct expression should be: -

The population of London is greater than that of any other city in India.

(a) When comparative degree is used with than, make sure that we exclude the thing compared from the rest
of class of things by using the

Example: He is stronger than any man living. (incorrect).

He is stronger than any other man living. (correct).

Similarly, Solomon was wiser than all other men.

In superlative degree, we must include the thing compared.

Solomon was the wisest of all men.

He is the strongest of all men.

Let‘s Practice 5: Like humans, who can recognize another‘s knowledge and beliefs—
an ability that may not be unique to mankind—reading others‘ intentions and displaying awareness of what
others see seem within the capabilities of some non-human primates.

A. reading others‘ intentions and displaying awareness of what others see seem within the capabilities of
some non-human primates.

B. the capabilities of some non-human primates include reading others‘ intentions and displaying awareness
of what others see.

C. some non-human primates seem capable of reading others‘ intentions and displaying awareness of what
others see.

D. some non-human primates' ability to read others‘ intentions and displaying awareness of what others
see.

E. non-human primates, reading others‘ intentions, seem to have some abilities of displaying awareness of
what others see.

9
Redundancy

This is the error of writing the same thing twice.

Example: a. He returned back from Delhi. B. I hardly have any money to give you.

The correct constructions should be;

a. He came back from Delhi. B. I have no money to give you.

Let‘s Practice 6: Many house builders offer rent-to-buy programs that enable a family with insufficient
savings for a conventional down payment to be able to move into new housing and to apply part of the rent
to a purchase later.

(A) programs that enable a family with insufficient savings for a conventional down payment to be able to
move into new housing and to apply

(B) programs that enable a family with insufficient savings for a conventional down payment to move into
new housing and to apply

(C) programs, which enable a family with insufficient savings for a conventional down payment to be able to
move into new housing, applying

Error of Participles

(Is the subject or doer close to the participle?)

Function of –ing verbs

What function do the –ing verbs do in the following sentences?

-Smoking is injurious to health. -Swimming is a good exercise. - He has given up


smoking.

⮚ They function as the subject or object in the sentence. If you ask the question ‗what is injurious or a
good exercise‘ or ‗what has he given up‘ you will get the –ing words as answer. You should ask
‗what‘ questions before the verb to know the subject.

⮚ While he was smoking, he entered the hall.


⮚ He entered the hall when he was smoking.

In the sentences above the words smoking and swimming don‘t answer the question ‗what entered or who
crossed‘ but tell how he entered or how they crossed.

These are different ways of combining two sentences.

Smoking is a participle here and the main function of a participle is to combine two sentences.

10
⮚ The two sentences, in this case, are: He was smoking. At that very same time he entered the hall.
Or,

He entered the hall. At that very same time he was smoking.

⮚ Consider the other sentence: While swimming, they crossed the flooded river.
⮚ If we split the same, we can say: They swam. This is how they crossed the river.

However, we can‘t say the above sentences in the following manner:

Smoking the hall was entered by him. (Incorrect)

Swimming the flooded river was crossed by them. (Incorrect)

Consider the following sentences:

Writing a novel, his name became famous.

Having been a very good scientist, they made him the president.

⮚ Ask ‗who wrote the novel, his name or some person?‘ It was some person. The sentence, therefore,
should be ‗Writing a novel, he became famous‘.
⮚ In the second sentence, who was a scientist–they or the president? It was the president. Therefore,
the sentence should be ‗Having been a very good scientist, he was made president by them‘.
⮚ It is clear from the above discussion that the doer of the participle verb should not be distanced from
it or interrupted by any other subject, noun or verb.

Let‘s Practice 7: After she attended the career fair, many more resources were at Ankita‘s disposal,
including job boards, new contacts, and numerous books and pamphlets to help her improve her resume and
cover letter.

A) many more resources were at Ankita‘s disposal

B) at Ankita‘s disposal were many more resources

C) there were many more resources at Ankita‘s disposal

D) Ankita, at her disposal, had many more resources

E) Ankita had many more resources at her disposal

Verb Tenses

Sequence of Tenses Is the principle in accordance with which the tense of the verb in a subordinate
clause follows the tense of the verb in the principal clause.

1. When verb in Principal clause is in Present or Future tense, the verb in the dependent clause may
be in any tense (according to sense).

11
1. He says that he likes music. Or He will say that he likes music.

2. He says that he liked music. Or He will say that he liked music.

● 3. He says that he will work hard. Or He will say that he will work hard.

2. When the verb in the Principal clause is in the Past Tense, the verb in the Dependent clause must
also be in one or other of the four forms of the past Tense.

1. He said that he was late.

2. You said that you would help him.

3. He worked hard so that he might pass.

4. He said that you were working hard.

EXCEPTIONS:

3. If the Dependent clause states a universal truth or a habitual truth, a Past Tense in the Principal
clause is followed by Present Tense in the Dependent clause.

1. He said that the sun rises in the east.

2. You said that you are a vegetarian.

4. A Past Tense in the Principal Clause is followed by any tense in the Adverbial clause beginning
with ‘than’ as

● I valued your friendship more than (I value) his.

● He liked you more than I did (like you).

● He liked you more than I do (like you).

● I then saw him oftener than I see him now.

5. A Past tense in the Principal Clause may be followed by any tense in the adjectival clause as:

● I visited the place where he lives.

● I visited the place where he lived.

● I visited the place where he will live

12
Let‘s Practice 8: Shortly after their first expeditions to the region, the Spanish initiated several attempts to
subjugate the Maya, but it takes some 170 years before the Spanish established substantive control over all
Maya lands.

a) but it takes some 170 years

b) but they would take some 170 years

c) but it took some 170 years

d) but it was going to take some 170 years

13
Practice
Q1. The loss of a parent at a young age can traumatize a person for the rest of your life.

a. for the rest of yourself's life.

b. for the rest of his or her life.

c. for the rest of your life.

d. for the rest of your living.

e. on the rest of your life.

Q2. Although the basketball team used their star players, the match was lost.

a. Although the basketball team used their star players

b. Although the star players were used by the basketball team

c. The basketball team used its star players, although

d. Although the basketball team used its star players

e. The basketball team used their star players

Q3 John and Susan runs to the finish line as fast as possible.

a. are run

b. runs to the finish line

c. run to the finish line

14
a. having run to the finish line

b. running to the finish line

Q4. The boy, among many friends, celebrate his birthday in lavish style.

a. having been celebrated

b. is celebrated

c. celebrates

d. celebrate

e. is celebrant

Q5. Autocratic styles of leadership frequently annoys the people who must work under such leaders.

a. frequented annoying

b. frequent annoy

c. frequently annoy

d. frequent annoys

e. frequently annoys

Q6. Coming back to the farm, the gruesome scene was seen by everyone in the car.

a. everyone in the car saw the gruesome scene.

b. the gruesome scene being seen by everyone in the car.

c. the gruesome scene was seen by everyone at the car.

d. the gruesome scene was seen by everyone in the car.

e. the gruesome scene saw by everyone in the car.

Q7. Waiting for the crucial trial to begin, the anxiety Neil felt was almost overwhelming.

a. the anxiety almost overwhelmed Neil.

b. the anxiety being felt by Neil was almost overwhelming.

c. the anxiety Neil felt was almost overwhelming.

d. Neil felt almost overwhelmed with anxiety.

e. Neil's anxiety felt almost overwhelming.

15
Q8. The vineyards of Napa Valley are at once breathlessly vibrant, symmetrical, and the green is profound.

a. and profoundly green.

b. and the green is profound.

c. and so profound green.

d. and it is profoundly green.

e. and its green is quite profound.

Q9. The teacher sought new ways of teaching, believing her students would get more from lessons featuring
playing, acting, and creativity.

a. featuring playing, acting, and creativity.

b. featuring playing, acting, and creating.

c. featuring playing, acting, and creative things.

d. featuring play, act, and creativity.

e. featuring the playing, acting, and creativity.

Q10. Stretching before physical workouts is important to improve flexibility and avoiding injury.

a. is important to improve flexibility and being injured less.

b. is important to both improve flexibility and also avoid injury.

c. is important to improve flexibility and avoid injury.

d. is important to improve flexibility and in the avoidance of injury.

e. are important to improve flexibility and avoiding injury.

Q11. Darren spent most of the day sulking alone in his room, but however he came out once to eat dinner
with his parents. No error

a. No error

b. to eat

c. most of the day

d. but however

e. spent

16
Q12. In addition to providing lunch for the children, the camp counselors also gave them dessert
and played educational games with them. No error:

a. them

b. No error

c. also

d. the children, the

e. played

Q13. Intelligence is a trait that is quite difficult to measure, despite much different attempts to do so.

a. despite the much different attempts to do so.

b. despite much different attempts to do it.

c. despite much different attempts to do so.

d. despite many different attempts to do so.

e. despite much different attempting to do so.

Q14. Under normal circumstances, the family would have never driven up to the frighteningly looking
motel.

a. for the frighteningly looking motel.

b. to a frighteningly looking motel.

c. to the frighteningly looking motel.

d. to the frightening looking motel.

e. to the frighteningly looked motel.

17
Q15. Questions abounded in the inquiry into illegal payments alleged made by the college to student
athletes.

a. alleged made

b. alleged making

c. allegedly made

d. allegedly making

e. made alleged

Q16. If seriously mentally ill people do not receive medication, they can grow unable to support themselves,
become irrational, and perhaps even threatening the safety of themselves or others.

a. and perhaps even threatening

b. and may even threaten

c. and even a possible threat to

d. as well as possibly threatening

e. as well as a possible threat to

Q17. Two disabled children, one with crutches and the other one with a wheelchair, enters the class on
Monday.

a. the other one with a wheelchair, enters

b. the other one a wheelchair, enter

c. the other with a wheelchair, enters

d. the other with a wheelchair, enter

e. one with a wheelchair, enters

Q18. If I had the address, I would have delivered the package myself.

A. had the address,

B. had the address;

C. had the address-

D. had had the address;

E. had had the address,


18
Q19. Some of them burned out eons ago, the night sky is spotted with thousands of stars.

a. Some of them burned out eons ago, the night sky is spotted with thousands of stars.

b. Burned out eons ago, the night sky is spotted with many thousands of stars.

c. Thousands of stars, some of them burned out eons ago, are spotting the night sky.

d. The night sky is spotted with thousands of stars, some of which are burnt out eons ago.

e. The night sky is spotted with thousands of stars, some of them burned out eons ago.

Q20. The humidity, air pollution, and noise have affected the children of Maria less drastically than those of
her neighbor.

a. affected the children of Maria less drastically than those of

b. affected the children of Maria less drastically than

c. affected the children of Maria less dramatically than they have

d. dramatically affected the children of Maria less than

e. dramatically affected the children of Maria and

Q21. Poverty is too much with us and its presence across vast stretches of our country disturbed our
conscience.

A. disturbed our consciousness

B. disturb its conscience

C. disturbs our conscious

D. disturbs our conscience

E. No correction required

Q22. Unlike that of the French, who linger when they eat meals, Americans are so enamored of eating
quickly that they have a type of meal called ―fast food‖.

a. Unlike that of the French, who linger when they eat


b. Unlike the French, who linger when they eat
c. Unlike the French, lingering when eating
d. Dissimilar to the French, lingering during
e. Lacking similarity to the French, who linger during

Q23. Certain painkilling drugs such as Oxycotin have recently been shown to be addictive to patients,
which may limit their potential to reduce pain.

a. which may limit their potential to reduce


19
b. which may limit their potential for reducing
c. which may limit such drugs‘ potential to reduce
d. an effect that may limit their potential to reduce

20
a. an effect that may limit the potential of such drugs to reduce

Q24. Books to be added to the high school curriculum should be educational and should have no
profanity in them or be lewd.

a. and should have no profanity in them or be lewd


b. and should not have profanity in them or not be lewd
c. and contain no profanity or lewdness
d. without containing profanity nor be lewd
e. without having any profanity or no lewdness in them

Q25. After Georgio‘s Caf˝O got a favorable review in a travel guidebook, the number of tourists eating
there were in excess of the number of local customers going regularly.

a. were in excess of the number of local customers


b. had an excess over the local customers who were
c. exceeded the local customers who were
d. numbered more than the local customers
e. exceeded the number of local customers

Q26. All the matches of Cricket World Cup will be broadcasted on Start Cricket.

A. will broadcast

B. are going to broadcast

C. going to be broadcasted

D. will be broadcast

E. No correction required

Q27. Montreal, where the tourist industry is larger than any other Canadian city, has neighborhoods entirely
composed of souvenir shops and cafes.

a. where the tourist industry is larger than any other Canadian city
b. which has a tourist industry larger than that of other Canadian cities
c. which had a tourist industry larger than any other Canadian city
d. whose tourist industry is larger than any other Canadian city
e. whose tourist industry is larger than that of any other Canadian city

Q28. Because Albert is the most experienced and he is therefore the best ballet dancer in the company, he is
being increasingly viewed by the director as the best candidate for the role of the Nutcracker.

a. and he is therefore the best ballet dancer in the company, he is being increasingly viewed
b. he is therefore the best of ballet dancers, and it has increased the view
c. and therefore, the best ballet dancer, he is being increasingly viewed
d. and therefore, he is the best of ballet dancers, there is an increasing view
e. therefore, being the best of ballet dancers, it is increasingly viewed
21
Q29. He did not mention how old he was or what is his gender.
A. what was his gender
B. what gender of his was
C. what his gender was
D. what the gender of him is
E. No correction required

Q30. There is nothing quite depressing than low level of unemployment in India.
A. as depressing
B. this depressing
C. more depressing
D. as depressing as
E. No correction required

UNIT 2
VOICE & ACCENT
What is Accent?

22
The term accent has various meanings, but in speaking, an accent is an identifiable style of pronunciation,
often varying regionally or even socioeconomically. But our aim is to neutralize & globalize your accent.

An accent is a particular way of pronouncing a language. 'Warsh' for wash in Cajun Louisiana, 'New Yawk'
for New York among native New Yorkers, 'aboot' for about in Canada. The appeal of dialects and accents
comes from our appreciation of their musical intonations, imaginative word choices, and emotive speech
rhythms.

Components of Accent

There are 2 main parts to an accent:

1. Intonation
2. Pronunciation

Some important points to know about English pronunciation:

23
● When we talk, we use our vocal organs – parts of our mouth and throat, such as our lips, teeth,
tongue and larynx (voice box). This whole area of the body is called the vocal tract.

● To make a vowel sound, we position the vocal organs further apart. The vocal tract is more open. For
example, the word I is pronounced as a vowel sound (the same one as the second sound in pie).

What are VOWELS?

Speech sounds are broadly categorized as Vowels & Consonants. Vowels are those sounds during the
utterance of which the air escapes freely, without any obstruction. In written English there are 5 vowels
i.e., a, e, i, o, u.

But, in spoken English there are 20 distinctive vowel sounds, made up of 12 pure vowels or
monophthongs and 8 vowel glides or diphthongs.

Definition: Vowel sounds are sounds that are produced by the un-interrupted flow of air.

Diphthongs: Diphthongs are 'gliding vowels', where one vowel sound glides into another one, as a
result of the lips or tongue moving. A diphthong is a change in vowel quality, whose sound
changes within the same syllable.

Monophthong: A vowel with a single sound quality, such as the middle sound in "rat" or "bit". These
sounds are made with one tongue position.

24
EXAMPLES OF SHORT AND LONG VOWEL SOUNDS

A: short A: long A: short A: long


back bake can cane
snack snake plan plain/plane
fad fade tap tape
mad made/maid at ate
Sam same cat Kate

E: short E: long E: short E: long


bed bead bet beat/beet
Ben bean met mete/meat/meet
men mean pet Pete
ten teen set seat

I: short I: long I: short I: long


lick like fin fine
hid hide shin shine
slid slide bit bite
dim dime lit light/lite
Tim time sit sight/site

O: short O: long O: short O: long


rob robe cot coat
hop hope not note
mop mope rot rote/wrote

U: short U: long U: short U: long

cub cube cute


cut
tub tube flute
flutter
hug huge mute
mutter

25
What are CONSONANTS?

Consonant sounds are produced by partial or full interruption of the breath flow. Consonants give
clarity and sharpness to the words. They do the work of making our speech crisp and clear, just like the
function of treble in a music system. When you whisper it is consonants that carry forward the sound and
the meaning.

‐ They also convey logic

A-Z is the Alphabet

B C D are called letters of the alphabet

The English Consonant sounds are divided into two broad buckets, which are
● Voiced Consonants-Plosive
● Unvoiced Consonants- Non-Plosive

Consonant sounds can be produced either with or without a vibration of the vocal

Unvoiced Voiced

/p/ path /b/ bath


/t/ time /d/ dime
/k/ came /g/game
/f/ fan /v/ van
/th/ think /th/them
/s/ price z/prize
/sh/shoe /zh/usual
/ch/chin /dj/gin

26
27
Voice drills

Articulation 1:
Repeat clearly:
● ba, ba, ba, wa, wa, wa, ba, ba, ba, wa, wa, wa, pu, pu, pu, fu, fu, fu, pu, pu, pu, fu, fu, fu
● Gutta butta, gutta butta, butta gutta, butta gutta, …
● Red leather, yellow leather, Red leather, yellow leather, …

Articulation 2:
Repeat clearly, and increase the speed at each repetition:
Bah dah gah pah dah gah
Boh doh goh poh doh goh
Boo doo goo poo doo goo
Bee dee gee pee dee gee
Bay day gay pay day gay

Articulation 3:
Repeat clearly, and increase the speed at each repetition:
Mah nah lah Thah vah zah
Moh noh loh Thoh voh zoh
Moo noo loo Thoo voo zoo
Mee nee lee Thee vee zee
May nay lay Thay vay zay

Articulation 4:
Sah Kah She Fah Rah
Pah Kah She Fah Rah
Wah Kah She Fah Rah
Bah Kah She Fah Rah
Dah Kah She Fah Rah

Articulation 5:
The tip of the tongue, the teeth and the lips
Lah lee loo lee. Zip e do da
Repetition, repetition, repetition
We'll weather the weather whatever the weather whether we like it or not

28
Activity: Try speaking these tongue twisters and differentiate amongst the vowel sounds.

1. She sells seashells by the seashore


2. I have got a date at a quarter to eight; I’ll see you at the gate, so don’t be late
3. I saw a kitten eating chicken in the kitchen
4. Eddie edited it
5. So, this is the sushi chef

6. Betty bought a bit of butter, but the bit of butter was bitter, so she bought another
bit of better butter to make the bitter butter better.

7. Eleven benevolent Elephants.

8. Six sticky skeletons

9. Friendly fleas and fireflies.

10. Red lorry, yellow lorry

29
Stress & Intonation

Syllables

Syllables are sound units that build up the structure of every word. They are a very important part of speech.
They give a word its pronunciation. Without them speech would sound dull, boring and meaningless.

Syllable is a unit of pronunciation having one vowel sound, with or without surrounding consonants,
forming the whole or a part of a word; for example, there are two syllables in water and three in inferno.

Syllable Stress is very important to understand as using stress on the wrong syllable can affect our
pronunciation.

Example: Develop Photographer Industry Technology

1. Two-Syllable nouns and adjectives

In most two syllables nouns and adjectives, the first syllable takes on the stress.

Examples:

SAMples CARton PURple RAIny CHIna HAPpy

2. Two-Syllable verbs and prepositions

In most two syllables verbs and prepositions, the stress is on the second syllable.

Examples:

reLAX diRECT aMONG aSIDE betWEEN deCIDE

30
More about word stress on two-syllable words

● About 80% of two-syllable words get their stress on the first syllable.
● There are, of course, exceptions to this rule, but very few nouns and adjectives get stress on their
second syllable.
● Verbs and prepositions usually get stress placed on the second syllable, but there are exceptions to
this too.

Note: There are many two-syllable words in English that can be pronounced in two different ways. The
stress change also changes the part of speech of the word.

Examples:

● PREsent = a gift (noun); non past or future (adjective)

● preSENT = to give something to someone (verb)

● OBject = something you can see and touch (noun)

● obJECT = to disagree with something (verb)

3. Three-Syllable words

For three-syllable words, word stress in on 2nd syllable most of the times. However, exceptions are always
there.

adEquate amAzement attEntion attrActive banAna

4. Words ending in er, ly

For three-syllable words ending with the suffixes er or ly, the stress is placed on the first syllable.

ORderly SIlently LOvingly MAnager GARdener EAsier

5. Words ending in consonants and in y

If there is a word that ends in a consonant or in a y, then the first syllable usually gets the stress.

RARity OPtimal GRAdient GEnorous

6. Words with various endings

Take a good look at the list of suffixes below (suffixes are word endings). The stress is going to be on the
syllable right before the suffix. This applies to words of all syllable lengths.

31
Examples:

1. able: ADDable, DURable, LAUGHable

2. ial: differENTial, SOcial, fiNANcial

3. cian: muSIcian, phySIcian, cliNIcian

4. ery: BAkery, SCEnery

5. ian: coMEdian, ciVILian, techNIcian

6. ible: reSIstible, imPOSsible, TERRible

7. ic: arCHAic, plaTOnic, characteRIStic

8. ics: diaBEtics, paediAtrics, TOpics

9. ion: classifiCAtion, repoSItion, vegeTAtion

10. ia: MEdia, bacTERia, vicTORia

11. ient: inGREdient, PAtient, ANcient

12. ious: mySTERious, reLIgious, VARious

13. ish: SELfish, ENglish, PUnish

14. osis: hypNOsis, diagNOsis, osMOsis

7. Words ending in ade, ee, ese, que, ette, oon

Words that use the suffix ade, ee, ese, eer, que, ette, or oon have the primary stress placed on the suffix.

1. ade: lemoNADE, cruSADE, arCADE

2. ee: aGREE, jamborEE, guaranTEE

3. eer: sightSEER, puppeTEER

4. ese: SiamESE, JapanESE, chEESE

5. ette: cassETTE, CorvETTE, towelETTE

6. que: unIQUE, physIQUE

7. oon: baLOON, afterNOON, carTOON

8. Stress on the second from the end syllable

32
You put stress on the second syllable from the end of the word with words ending in ic, sion, and tion.

iCONic GRAPHic hyperTENsion teleVIsion nuTRItion

9. Stress on the third from end syllable

You put stress on the third from end syllable with words that end in cy, ty, phy, gy and al.

deMOcracy geOGraphy ALlergy CLArity


CRItical

10. Word stress for compound words

A. Compound noun

A compound noun is a noun made from two nouns that form one word. In a compound noun, the most stress
is on the stressed syllable of the first word.

● SEAfood (sea + food)

● ICEland (ice + land)

● TOOTHpaste (tooth + paste)

● FOOTball (foot + ball)

● BAsketball (basket + ball)

B. Compound adjectives

A compound adjective is an adjective made of at least two words. Often, hyphens are used in compound
adjectives. In compound adjectives, the most stress is placed in the stressed syllable of the second word.

● ten-MEter

● rock-SOlid

● fifteen-MInute

● old-FAshioned

C. Compound verbs

33
A compound verb is when a subject has two or more verbs. The stress is on the second or on the last part.

● Matilda loves bread but deTESTS butter.

● Sarah baked cookies and ATE them up.

● Dogs love to eat bones and love to DRINK water.

D. Noun + compound nouns

Noun + compound Nouns are two-word compound nouns. In noun + compound noun, the stress is on the
first word.

● AIRplane mechanic

● PROject manager

● BOARD member

11. Reflexive pronouns

Reflexive pronouns show that the action affects the person who performs the action. For example: I
hit mySELF. The second syllable usually takes the stress.

● mySELF

● themSELVES

● ourSELVES

What is Sentence Stress?

Words in a sentence are not all given the same salience (importance) in oral English. Some words are picked
out and are stressed in contrast to others. The one that is the most stressed is said to receive the sentence
stress. This usually implies differences in meaning.

I did not say you stole my red hat.

At this moment, nothing is particularly stressed. The meaning seems obvious. But what if some stress is
placed on the first word -

I: I did not say you stole my red hat.

Then the meaning contains the idea that someone else said it, not me. Stress the second and third word and
you get another shade of meaning:

I did not say you stole my red hat. (Strong anger and denial of the fact.)

I did not say you stole my red hat. (Strong anger and denial of the fact.)
34
I did not say you stole my red hat. (But I implied it that you did. Did you?)

I did not say you stole my red hat (I wasn't accusing you. I know it was someone else)

I did not say you stole my red hat. (I said you did something else with it, or maybe borrowed it.)

I did not say you stole my red hat. (I meant that you stole someone else's red hat)

I did not say you stole my red hat. (I said that you stole my blue hat.)

I did not say that you stole my red hat. (I said that you stole my red bat. You misunderstood my
pronunciation)

Analyzing this way, you can see how important stress is in English. Now, you need to understand which
words we generally stress and which we do not stress.

Most sentences have two basic types of word:

● Content words: Content words are the key words of a sentence. They are the important words that
carry the meaning or sense—the real content.

● Structure words: Structure words are not very important words. They are small, simple words that
make the sentence correct grammatically. They give the sentence its correct form—its structure.

If you remove the structure words from a sentence, you will probably still understand the sentence. If you
remove the content words from a sentence, you will not understand the sentence. The sentence has no sense
or meaning.

35
Imagine that you receive this telegram message:

This sentence is not complete. It is not a "grammatically correct" sentence. But you probably understand it.
These 4 words communicate very well. Somebody wants you to sell their car for them because they
have gone to France. We can add a few words:

But the information is basically the same:

VOICE MODULATION (INTONATION)

Intonation

Intonation is the music of the language. In English, we use tone to signal emotion, questioning, and parts of
the sentence among many other things. It's important to recognize the meaning behind the tones used in
everyday speech, and to be able to use them so that there are no misunderstandings between the speaker and
the listener. It is generally true that mistakes in pronunciation of sounds can be overlooked, but mistakes in
intonation make a lasting impression.

In other words, we can say that the intonation of a language refers to the patterns of pitch variation, or
the tones, its uses in its utterance.

Falling Intonation (➘) (The pitch of the voice falls at the end of the sentence.) Falling intonation is
the most common intonation pattern in English. It is commonly found in statements, commands, wh-
questions (information questions), confirmatory question tags and exclamations.

● Statements

o Nice to meet ↘ you.

o I‘ll be back in a ↘ minute.

o She doesn‘t live here ↘ anymore.

o Dad wants to change his ↘ car.

o Here is the weather ↘ forecast.

o Cloudy weather is expected at the end of the ↘ week.

36
● Commands

o Write your name ↘ here.

37
o Show me what you‘ve ↘ written.

o Leave it on the ↘ desk.

o Take that picture ↘ down.

o Throw that ↘ out.

o Put your books on the ↘ table.

o Take your hands out of your ↘ pockets.

● Wh- questions (requesting information.)

(Questions beginning with 'who', 'what', 'why', 'where', 'when', 'which', and 'how')

o What country do you come ↘ from?

o Where do you ↘ work?

o Which of them do you ↘ prefer?

o When does the shop ↘ open?

o How many books have you ↘ bought?

o Which coat is ↘ yours?

o Whose bag is ↘ this?

● Questions Tags that are statements requesting confirmation rather than questions. Not all tag
questions are really questions. Some of them merely ask for confirmation or invite agreement, in
which case we use a falling tone at the end.

o He thinks he‘s so clever, doesn‘t ↘ he?

o She's such a nuisance, isn't ↘ she?

o I failed the test because I didn't revise, did ↘ I?

o It doesn't seem to bother him much, does ↘ it?

● Exclamations

o How nice of ↘ you!

o That's just what I ↘ need!

o You don't ↘ say!

38
o What a beautiful ↘ voice!

o That's a ↘ surprise!

Rising Intonation (➚) (The pitch of the voice rises at the end of a sentence.)

Rising intonation invites the speaker to continue talking.


It is normally used with yes/no questions, and question tags that are real questions.

● Yes/no Questions

● (Questions that can be answered by 'yes' or 'no'.)

o Do you like your new ➚teacher?

o Have you finished ➚already?

o May I borrow your ➚dictionary?

39
o Do you have any ➚magazines?

● Questions tags that show uncertainty and require an answer (real questions).

o We've met already, ➚haven't we?

o You like fish, ➚don't you?

o You're a new student ➚aren't you?

o The view is beautiful, ➚isn't it?

We sometimes use a combination of rising and falling intonation in the same sentence. The
combination is called Rise-Fall or Fall-Rise intonation.

Rise-Fall Intonation (➚➘) (The intonation rises and then falls.)

We use rise-fall intonation for choices, lists, unfinished thoughts and conditional sentences.

● Choices (alternative questions.)

o Are you having ➚soup or ➘salad?

o Is John leaving on ➚Thursday or ➘Friday?

o Does he speak ➚German or ➘French?

o Is your name ➚Ava or ➘Eva?

● Lists (rising, rising, rising, falling)

● Intonation falls on the last item to show that the list is finished.

o We've got ➚apples, pears, bananas and ➘oranges

o The sweater comes in ➚blue, white pink and ➘black

o I like ➚football, tennis, basketball and ➘volleyball.

● Unfinished thoughts (partial statements)

In the responses to the following questions, the rise-fall intonation indicates reservation. The speaker
hesitates to fully express his/her thoughts.

40
o Do you like my new handbag? Well, the ➚leather is ➘nice... (but I don't like it.)

o What was the meal like? Hmm, the ➚fish was ➘good... (but the rest wasn't great).

● Conditional sentences

(The tone rises in the first clause and falls gradually in the second clause.)

o If he ➚calls, ask him to leave a ➘message.

o Unless he ➚insists, I'm not going to ➘go.

Practice:

1. Insert the correct word(s) in the sentences below.

a) If we go __________ the beach, would you like to come __________? (to/ too/ two)

41
a) I am going __________ you like it or not. (whether/ weather)
b) I shall put __________ parcels over __________. (there / their/ they‘re)
c) I like to come __________ because I always __________ the latest gossip. (hear/here)
d) There is __________ paper left in the printer, or did you __________ that already? (no/ know)
e) __________ you like to come to the party with me? (would/ wood)
f) You need to go __________ the door at the end of the corridor to get to the exit. (through/ threw)
g) I must __________ a letter to the bank. (write / right)
h) I need to see if he has cashed the __________. (check/ cheque)
i) He __________ his motorbike along the __________. (road/ rode/ rowed)

2. Fill in the blanks using ‗aw‘ sound.

a. Niagara _ ___ls is the highest falls in the world.


b. The children are playing with the basket __ ____.
c. He got a ______ _____ship from his school.
d. ____ _____ walk to your left on the road.
e. Roses have _ ______.

3.Unscramble the jumbled words


d) Oyj -
a) Liob - e) Inoc -
b) Jyeon - f) Sybo -
c) Snioe - g) Ayorl -

4.Choose the Correct Response.

a. I went to the barber to get a (hair/ hare) cut.


b. Sheena is a (fare/ fair) girl.
c. On Ridhima‘s birthday her father gifted her a teddy (bare/ bear).
d. Please don‘t (stare/ stair) at the stranger.
e. I bought a (pear/ pair) of scissors for the craft classes.
f. I will (ware/ wear) a red dress for the party.

5. Use ‗wear‘, ‗where‘ or ‗we‘re‘ correctly:

a) Make sure you __________________ a life jacket on the boat.


b) Please put the art supplies back ____________________ they belong.
c) ____________________ going on a road trip next Friday.
d) Mom told me to __________________ a coat, but I didn‘t listen.
e) I don‘t know ____________________ I put those moving boxes.
f) This weekend, __________________ going to the park to play baseball.
g) _____________________ will you hide the gifts?
h) My sister wants to _____________ the sweater she got for her birthday.
i) John invited us to breakfast, but ______________ not going to make it on time.
j) 10.I will ________________ my red hat to school tomorrow.

42
6. In English, many written words contain consonant letters that are not pronounced. These letters are
referred to as ‗silent‘ letters. Match each word with the appropriate meaning below.

Rhyme often listen island calm column talk


foreign hour exhausted bark

a) 1.sixty minutes __________________________


b) 2land surrounded by water __________________________
c) 3 peaceful and quiet __________________________
d) 4 words containing the same sounds __________________________
e) 5 to hear and give attention when someone speaks __________________________
f) 6 to speak __________________________
g) 7 to be very tired and without energy __________________________
h) 8 from another country or another place __________________________
i) 9 a strong, tall piece of stone or wood __________________________
j) 10 used to support a building __________________________

7. Read the sentence. Write the correct word on the line.

a. The __________________ going around the mountain is very scary. (Rode, road)
b. _____________________ not going to believe what I saw today. (Your, you‘re)
c. I don‘t know if I would like ___________________ soup or not. (Beet, beat)
d. Stanley ran around two ____________________ before they threw him out. (Bases, basis)
e. The huge _________________________ sat dangerously close to the edge. (bolder, boulder)
f. The ______________________ in our new house will be ten feet tall. (ceiling, sealing)
g. The _______________________ ripped on the curtain when the cat climbed it. (seem, seam)
h. My neighbor delivers the ______________ on your ____________. (mail, male) (rode, road)
i. He said the dog was __________________ but __________ not. (theirs, there‘s) (its, it‘s)
j. My mom bought us four different kinds of __________________________ to eat. (cereal,
serial)
k. An independent _________________________ is a sentence. (clause, claws)
l. We rented a hotel ___________________ when we went on vacation. (suite, sweet)

8. Write sentences for 2 pairs of words, demonstrating the different word class:

1. Conduct

Noun
……………………………………………………………………………………………………………………
………………………………………………………………………………………………………………

Verb
……………………………………………………………………………………………………………………
………………………………………………………………………………………………………………

2. Digest
43
Noun
……………………………………………………………………………………………………………………
………………………………………………………………………………………………………………

Verb
……………………………………………………………………………………………………………………
………………………………………………………………………………………………………………

3. Escort

Noun
……………………………………………………………………………………………………………………
………………………………………………………………………………………………………………

Verb
……………………………………………………………………………………………………………………
………………………………………………………………………………………………………………

4. Insult

Noun
……………………………………………………………………………………………………………………
………………………………………………………………………………………………………………

Verb
……………………………………………………………………………………………………………………
………………………………………………………………………………………………………………

5. Produce

Noun
……………………………………………………………………………………………………………………
………………………………………………………………………………………………………………

Verb
……………………………………………………………………………………………………………………
………………………………………………………………………………………………………………

6. Record

Noun
……………………………………………………………………………………………………………………
………………………………………………………………………………………………………………

Verb
……………………………………………………………………………………………………………………
………………………………………………………………………………………………………………

7. Access

44
Noun
……………………………………………………………………………………………………………………
………………………………………………………………………………………………………………

Verb
……………………………………………………………………………………………………………………
………………………………………………………………………………………………………………

8. Address

Noun
……………………………………………………………………………………………………………………
………………………………………………………………………………………………………………

Verb
……………………………………………………………………………………………………………………
………………………………………………………………………………………………………………

9. Auction

Noun
……………………………………………………………………………………………………………………
………………………………………………………………………………………………………………

Verb
……………………………………………………………………………………………………………………
………………………

10. Balance

Noun
……………………………………………………………………………………………………………………
………………………………………………………………………………………………………………

Verb
……………………………………………………………………………………………………………………
………………………………………………………………………………………………………………

11. Bargain

Noun
……………………………………………………………………………………………………………………
………………………………………………………………………………………………………………

Verb
……………………………………………………………………………………………………………………
………………………………………………………………………………………………………………

12. Blame

45
Noun
……………………………………………………………………………………………………………………
………………………………………………………………………………………………………………

Verb
……………………………………………………………………………………………………………………
………………………………………………………………………………………………………………

13. Blast

Noun
……………………………………………………………………………………………………………………
………………………………………………………………………………………………………………

Verb
……………………………………………………………………………………………………………………
………………………………………………………………………………………………………………

14. Catch

Noun
……………………………………………………………………………………………………………………
………………………………………………………………………………………………………………

Verb
……………………………………………………………………………………………………………………
………………………………………………………………………………………………………………

15. Cause

Noun
……………………………………………………………………………………………………………………
………………………………………………………………………………………………………………

Verb
……………………………………………………………………………………………………………………
………………………

16. Convict

Noun
……………………………………………………………………………………………………………………
………………………………………………………………………………………………………………

Verb
……………………………………………………………………………………………………………………
………………………………………………………………………………………………………………

17. Complex

46
Noun
……………………………………………………………………………………………………………………
………………………………………………………………………………………………………………

Verb
……………………………………………………………………………………………………………………
………………………………………………………………………………………………………………

18. Desert

Noun
……………………………………………………………………………………………………………………
………………………………………………………………………………………………………………

Verb
……………………………………………………………………………………………………………………
………………………………………………………………………………………………………………

19. Segment

Noun
……………………………………………………………………………………………………………………
………………………………………………………………………………………………………………

Verb
……………………………………………………………………………………………………………………
………………………………………………………………………………………………………………

20. Content

Noun
……………………………………………………………………………………………………………………
………………………………………………………………………………………………………………

Verb
……………………………………………………………………………………………………………………
………………………………………………………………………………………………………………

9. Read the sentence. Write the correct word on the line:

a. ____________________ going to be the first to recite ___________________ poem? (Whose,


Who‘s) (their, there, they‘re)
b. The window ____________________ needs to be painted white. (pain, pane)
c. The yellow part of an egg is called the _______________________, (yoke, yolk)
d. Cindy makes ____________________ salad every time I eat at her house. (pare, pair, pear)
e. Grandma taught me how to _________________ bread __________________. (need, knead)
(doe, dough)
f. I love going to the county _______________________ each year. (fare, fair)
g. My dad always says, ―I love you _____________‖ to my mom. (deer, dear)

47
a. You need to make up your ___________________ before _________ too late. (mined, mind)
(its, it‘s)
b. They are going to _____________________ Tuesday with __________________ friends.
(meat, meet) (there, their, they‘re)
c. The dress looks much better with a belt around the __________________. (waist, waste)
d. Martha Steward showed us how she ____________________________ her turkey. (trust,
trussed)
e. Melvin ____________________ make that mistake again. (want, won‘t)

10. How many syllables are there in each word? Choose the correct answer.

1. monkey

a) 1 b) 2 c) 3 d) 4

2. relocation

a) 1 b) 2 c) 3 d) 4

3. magnet

a) 1 b) 2 c) 3 d) 4

4. slobs

a) 1 b) 2 c) 3 d) 4

5. characterize

a) 1 b) 2 c) 3 d) 4

6. travelling

a) 1 b) 2 c) 3 d) 4

7. rocket

a) 1 b) 2 c) 3 d) 4

8. trying

a) 1 b) 2 c) 3 d) 4

9. garden

a) 1 b) 2 c) 3 d) 4

10. Caterpillar

48
a) 1 b) 2 c) 3 d) 4

11. Where is the stress in each of these words? Decide which syllable:

1. reception
a) 1st b) 2nd c) 3rd

2. comparison

49
a) 1st b) 2nd c) 3rd d) 4th

3. potato

a) 1st b) 2nd c) 3rd

4. bedroom
a) 1st b) 2nd

5. fourteen
a) 1st b) 2nd

6. forty
a) 1st b) 2nd

7. delicious
a) 1st b) 2nd c) 3rd

8. playful
a) 1st b) 2nd

12. Use underlining to show the correct stress on these compound words which have been given in a
sentence to make the word class obvious:

a. I heard a blackbird singing.


b. Put the seedlings in the greenhouse until they are taller.
c. He is a bad-tempered old man.
d. My grandparents are a little old-fashioned. (2 words)
e. I don‘t understand what you mean.
f. The water will overflow and come out through this overflow pipe. (2 words)
g. I need to go to the supermarket before I leave for the airport. (2 words)
h. He is waiting at the bus-stop on the main highway. (2 words)
i. He came straight out of the swimming-pool and into the living-room to answer the telephone.
(3 words)
j. This raincoat isn‘t waterproof. (2 words)
k. Traffic-lights are confusing because I‘m colour-blind. (2 words)
l. What‘s he like? Well, he‘s easy-going, and good-looking, very self-confident and always well-
dressed. (4 words)

13. For each question, the correct choice is the one in which the stressed syllable is capitalized, as in
vocabulary:

1. Can you pass me a plastic knife?


a) PLAS-tic
b) plas-TIC

2. I want to be a photographer.

50
a) PHO-to-graph-er
b) pho-TO-graph-er

3. Which photograph do you like best?


a) PHO-to-graph
b) pho-TO-graph

4. He was born in China.


a) CHI-na
b) Chi-NA

5. Whose computer is this?


a) com-PU-ter
b) com-pu-TER

6. I can't decide which book to borrow.


a) DE-cide
b) de-CIDE

7. Couldn't you understand what she was saying?


a) un-DER-stand
b) un-der-STAND

8. Voting in elections is your most important duty.


a) im-POR-tant
b) im-por-TANT

9. We had a really interesting conversation.


a) con-VER-sa-tion
b) con-ver-SA-tion

10. How do you pronounce this word?


a) PRO-nounce
b) pro-NOUNCE

14. Match the following conversations with the correct sentence stress:

A. Where did you get these flowers from? The cemetery? 1. I asked you to buy me a bunch of white
roses.
B. Yes....

A. You weren‘t supposed to steal them!


________________

51
A. Here are the flowers Bob asked me to get. 2. I asked you to buy me a bunch of white
roses.
B. Eh? Bob didn‘t say anything, _______________

A. Why do these roses have your mother‘s name on 3. I asked you to buy me a bunch of white
them? roses.

B. I got them for my mother, just like you asked.

A. In what world would I ask you to buy your mother


flowers on our anniversary? _______________

A. John, why are there yellow roses on the table? 4. I asked you to buy me a bunch of white
_______________ roses.

A. Oh, lilies, they‘re beautiful, but _____________ 5. I asked you to buy me a bunch of white
roses.

A. I was kind of busy today, so my secretary did me the 6. I asked you to buy me a bunch of white
favour of ordering you the flowers you wanted. roses.

B. What?! ____________________ Why do I want roses


from your secretary?

A. Listen, I know a single rose is supposed to be very 7. I asked you to buy me a bunch of white
romantic, but ________________ roses.

A. Susie, you‘re always bossing me around. Ordering me 8. I asked you to buy me a bunch of white
to buy you flowers is the last straw. roses.

B. Fred, honestly, I don‘t understand what you‘re talking


about. __________________, not order!

15. Look at each of these words. Decide on which syllable the stress falls?

1. personal
a. 1st b. 2nd c. 3rd
2. personnel
a. 1st b. 2nd c. 3rd
3. adjective
a. 1st b. 2nd c. 3rd

52
4. enhance
a. 1st b. 2nd
5. Canadian
a. 1st b. 2nd c. 3rd d. 4th
6. Japanese

53
a. 1st b. 2nd c. 3rd
7. psychology
a. 1st b. 2nd c. 3rd d. 4th
8. politician
a. 1st b. 2nd c. 3rd d. 4th

Pronounce the following words using the correct knowledge of stress and syllables?

Entrepreneur
Engineer
Workaholic
Empathetic
Et cetera
Jewellery
Often
Phenomenon
Restaurant
Draught
Mischievous
Colonel
Subtle
Dessert
Vegetable

54
UNIT 3

Vocabulary

55
Enhancing vocabulary is like enhancing one ‗s life. There are a whole lot of experiences that one gets when
one learns new words. It opens new vistas as one comes to know the history & geography of the word and the
background associated with the word like the culture and mannerism of the place where the word is often
used.

There are various ways by which one can improve one‘s vocabulary. We will be discussing some of them
here:

A. Cloze test
What is a cloze test?

In a cloze test, one will be given a paragraph in which some of the vocabulary words will be omitted.

One will have to choose the appropriate word from a given list.

For example:

Read the paragraph to find the missing words:

A spider creates the web, to trap other creatures on whom it preys for (1) __________. A web in an
ecological community is a succession of organisms that are linked to each other through the transfer of energy
and nutrients. Human history is (2) _________ with stories of webs of (3) _________ in royal families or
business houses. The world wide web, which started as a means of military communication is now our lifeline
– it makes communication, entertainment and education possible through a web of invisible waves that enable
the exchange of information.

1. a. Sustenance b. jeopardy c. trepidation d. animosity


2. a. devoid b. replete c. engulfed d. mesmerized
3. a. intrigue b. overcoming c. pseudonyms d. embankment
Now in this question, one needs to find the missing three blanks with words from choices which are
appropriate in the context.

 In the first blank, the food of the spider is being talked about.

56
So, the correct word will be sustenance.

 In the second blank, the word that should fit in the context will be that the human history has many
examples of stories of webs.

So, the correct related word will be ―replete‖.

In fact, this is a tip to solve these kinds of questions:

That is to think of a word which fits into the blank without looking at the options. This will prevent one from
any confusion.

 For the third blank, one must go through all the options first for answering the question.

So here is another tip:

There is no one size fits all approach for solving these questions. It depends upon the question and what
strategy fits best like one could go for the elimination technique if one finds that there are two or more options
having the same meaning.

For example:

Thoughts are things and if we believe strongly enough, it __________universal forces to actually make things
happen.

a. Unleashes
b. Hinders
c. Prohibits
d. Obstructs
Here it can be seen that 3 options (b, c, d) have the same meaning.

Therefore, all three can be eliminated which leaves us with option a as the correct answer.

Practice Questions:

1.1 Choose the best option for each blank.

Polar bears are in (1) ___________________ of dying out. (2) ___________________ some other endangered
animals, it's not hunters that are the problem, it's climate change. Since 1979, the ice cap at the Arctic Circle
where polar bears live has (3) ___________________ in size by about 30 per cent. The temperature in the
Arctic has slowly been (4) ___________________ and this is (5) ___________________ the sea ice to melt,
endangering the polar bears' home. The polar bears' main (6) ___________________ of food are the different
types of seal found in the Arctic. They catch them by waiting next to the air holes the seals have (7)
___________________ in the ice. (8) ___________________ the bears are very strong swimmers; they could
never catch seals in the water. This means that the bears (9) ___________________ do rely on the ice to hunt.
Polar bears also need sea ice to travel. They can (10) ___________________ a huge territory and often swim
from one part of the ice to another. They have been (11) ___________________ to swim up to 100km, but
when there is less ice, they may have to swim further, and this can (12) ________________ fatal to the bears.
A number of bears have drowned in the last few years and scientists believe that it is because they were not
able to (13) ___________________ more sea ice before they became too tired and couldn't swim any further.
57
1 threat warning risk danger

2 Unlike Compared Opposite Different

3 reduced cut shortened lost

4 lifting rising gaining advancing

5 creating turning causing resulting

6 origins means materials sources

7 placed made put set

8 Despite As Although Even

9 properly surely fully really

10 cover spread extend pass

11 experienced noticed known taught

12 happen come end prove

13 reach land get Achieve

1.2 Choose the best option for each blank.

The Dead Sea is (1) ___________________ where the river Jordan ends, just 24 kilometers east of Jerusalem.
It is actually a lake. It (2) ___________________ for about 74 kilometers and is 16 kilometers wide, so it is
quite small but (3) ___________________ deep - about 300 meters. (4) ___________________, the Dead Sea
was about the same size as today. Then the climate of the area changed and became wetter. This change (5)
___________________ the Dead Sea to grow longer. However, after some time, the climate changed again,
and the lake returned to its original size and (6) ___________________. The Dead Sea is one of the saltiest
lakes in the world. One liter of seawater may (7) ___________________ up to 327 grams of salt. With so
much salt in the water, no fish or flora can survive in the Dead Sea. (8) ___________________, for people
who love swimming, it's the ideal place to (9) ___________________. And the salt in the water makes it easy
to swim without any (10) ___________________ at all! There is not much rainfall in this place of the world.
It usually rains only between October and March. The temperatures vary (11) ___________________ on the
area you look at. In the northern parts of the Dead Sea winter temperatures reach only about 14° C; however,
in August it can get up to 34 °C in the south. The highest (12) ___________________ temperature for this
region is 51 °C. Nearly twenty years ago there was a (13) ___________________ to build a canal between the
Dead Sea and the Mediterranean. This would make the (14) ___________________ of the water in the Dead
Sea go up. However, the cost of doing this is so (15) ___________________ that the project cannot go ahead
yet.

58
1 found situated set placed

2 ranges spreads covers extends

3 widely somewhat surely extremely

4 Originally Finally Firstly Eventually

5 led brought affected caused

6 outline shape nature structure

7 consist comprise contain exclude

8 Besides However Additionally Also

9 reduce ease calm relax

10 try sweat force effort

11 varying depending changing differing

12 recorded detailed taped proven

13 strategy design plan sketch

14 ground level point amount

15 enormous giant vast extensive

One-word Substitutions

One-word substitutions are a short way of saying a long thing. They help to say things in a precise and clear-
cut manner.

Below is a list of some important one-word substitutions:

59
2.1 Find a one-word substitute for the following:

1. An examination of tissue removed from a living body to discover the presence, cause or extent of a
disease
a. Autopsy
b. biopsy
c. histopathology
d. Necropsy

2. The arrangement of events or dates in the order of their occurrence


a. histogram
b. chronology
c. atemporal
d. Intermittent
60
3. A vigorous campaign for political, social, or religious change
a. entente
b. ceasefire
c. crusade
d. truce

4. Release someone from a duty or obligation


a. convict
b. exonerate
c. pacify
d. recapitulate

5. Summarize and state again the main points of.


a. propound
b. state
c. amplify
d. Recapitulate

6. A solution or remedy for all difficulties or diseases


a. euphoria
b. eureka
c. panacea
d. Hysteria

7. Excessively concerned with minor details or rules


a. philistine
b. lowbrow
c. pedantic
d. Conventional

8. Violation or misuse of what is regarded as sacred


a. violence
b. sacrilege
c. destruction
d. Mutiny

9. An imaginary ideal society free of poverty and suffering


a. Dystopia
b. utopia
c. sinecure

61
d. Photopia

10. In the same words as were used originally


a. verbatim
b. verbose
c. indirect
d. imprecise

11. The theory or philosophy of law


a. jurisprudence
b. jurisdiction

c. criminology

d. judiciary

12. The art of effective or persuasive speaking or writing


a. Impromptu speech
b. Rhetoric
c. Verbatim
d. Axiology

13. The Government wing responsible for making Rules


a. Judiciary
b. Executive
c. Court
d. Legislature

14. One who can use either hand with ease


a. ambivalent
b. Ambidextrous
c. Omnipotent
d. Cosmopolitan

15. A person appointed by two parties to solve a dispute


a. Mediator
b. Arbitrator
c. Middleman
d. Solvent

62
16. An unconventional style of living
a. Orthodox
b. Unorthodox
c. Bohemian
d. Brahman

17. A person who is blindly devoted to an idea/ a person displaying aggressive or exaggerated patriotism
a. Misogynist
b. Sexist
c. Chauvinist
d. Connoisseur

18. A leader or orator who espouses the cause of the common people
a. Crusader
b. Idealogue
c. Demagogue
d. Debonair

19. A lover of tasty food


a. Gourmet
b. Henpeck
c. Indefatigable
d. Highbrow

20. One who shows sustained enthusiastic action with unflagging vitality
a. Indefatigable
b. Iconoclast
c. Impregnable
d. Recluse

Sentence Synonyms

Synonyms have similar meaning words. For example:

The words ―juggernaut,‖ ―herculean‖, ―mammoth‖ and ―gigantic‖ all mean huge or big.

But all these words have different origins:

Juggernaut comes from the chariot of Lord Jagannath which starts from Puri, Odisha every year in India.

63
Herculean refers to ―Hercules,‖ the son of Jupiter in Greek Mythology.

Mammoth is the extinct species of Elephant.

64
So, these words may or may not be interchangeable within a sentence.

For example:

The Milky way is very gigantic. (Sentence is correct)

But it cannot be said that

The milky way is mammoth. (As the milky way is in fact much larger than any species of elephant).

So, the context in which the word is used or can be used is very important.

For practicing this aspect of vocabulary, sentence synonyms have been introduced wherein one has to fit in
another similar and appropriate word than the one already there in the sentence.

For example:

Choose a suitable word to replace the underlined word in the following sentence:

As the 20th century dawned, both China and India were gripped by powerful ideas of nationalism and
internationalism in the period between the two world wars.

a. compelling
b. Arbitrary
c. Neutral
d. Cognizant
The word that is synonymous with ―powerful‖ is compelling. Thus, it can be replaced with ―powerful‖ in the
sentence.

3.1 Practice Questions:

Replace the underlined word with a suitable word from the options given below:

65
1.Crony capitalism generates monopolies that reduce competition, strangle innovation and disincentivize
smaller businesses that create jobs and economic dynamism.

a. Choke
b. Encourage
c. Promote
d. Discourage

2.In the US, Facebook, a private company, denied the US President the use of a communication platform — a
testimony to the power of its digital empire.

a. Corroboration
b. Rebuttal
c. Refutation
d. Veto
3. Attacking inside an Air Force base with drones is certainly an escalation — in terms of targeting Indian
military assets.

a. Abridgement
b. Intensification
c. Amalgamation
d. Articulation
4.UAVs have now become far more potent with extensive capabilities.

a. Cogent
b. Infirm
c. Fragile
d. Disastrous
5.Drones can be used for reconnaissance, observation, as well as to carry small payloads, including mail and
parcels.

a. Reconnoitering
b. Reconsideration
c. repugnance
d. Recalcitration

6.The pandemic-induced learning crisis and the Fourth Industrial Revolution have made it necessary to
reimagine education and align it with the unprecedented technological transformation.
a. Unparalleled
b. Extraordinary
c. Miraculous
d. All the above
7.The pandemic threatens to exacerbate the learning crisis, especially because of the physical closure of 15.5
lakh schools that has affected more than 248 million students for over a year.

a. Alleviate
b. Soothe
c. Pacify
66
d. Aggravate
8.India‘s new National Education Policy (NEP) 2020 is responsive to the clarion call to integrate technology
at every level of instruction.

a. Inspiring
b. Low
c. Uninspiring
d. Cynical
9.India is well-poised to take this leap forward with increasing access to tech-based infrastructure, electricity,
and affordable internet connectivity, fueled by flagship programs such as Digital India.

a. Bellwether
b. Common
c. Rare
d. Recent
10.The US digital giant has been at loggerheads with the government over the new social media rules.

a. geniality
b. Disagreement
c. Conviviality
d. Affability

Learning vocabulary with images:

67
This is one of the best ways to improve vocabulary as pictures are always retained in the minds in comparison
to text. So, if one looks at the pictorial meaning of a word, it can easily be remembered and ingrained into the
vistas of the brain.

For example:

Let us look at the word ―Parochial‖

This word means ―having a limited or narrow outlook or scope.‖

Compare this with the image:

68
Doesn‘t that make a difference in understanding the word?

This is how vocabulary can be built and mastered?

Let us look at another example:

The word ―mammoth‖ means something huge.

Now if we were to remember this word by having the understanding that ―mammoth‖ is actually an extinct
species of an elephant found in the arctic regions which used to be very huge and hence the name became
synonymous with ―huge,‖ won‘t it be easier to remember the meaning of the word?

Practice questions
4.1

Identify the word with the help of image:

1.

69
a) contraction
b) Imagination
c) Workcation
d) Outstation

2. What word you can associate with the following image?

a) Lament
b) Embankment
c) Assortment
d) Attainment

3.What word you can associate with the following image?

70
a) Forbear
b) Renounce
c) Back drop
d) Clearance

4. What word you can associate with the following image?

a) breach

71
b) scenario
c) caretaker
d) flask
5.

a) Over confidant
b) Blasphemous
c) Inferiority complex
d) Sanctimonious

6.

A. Alacrity
72
B. Precarity
C. Tenacity
D. Serenity
7.

A. Ability
B. Overpopulation
C. Fecundity
D. Inability
8.

a) Serendipity

73
b) Surreptitiously
c) creatively
d) fastidiously
9.

a) Honesty
b) Masquerade
c) Vulnerability
d) Orthodoxy
10.

a) Suppression
b) Teamwork
c) Back biting
d) Friendly concern
11.

74
a) trajectory
b) Reckless
c) Using lofty speech
d) pretentious
Apply the meaning to a real-life situation

A word can tell an entire story. Words in fact contribute to the story telling process.

For Example:

Sangfroid

If one had to make a story with the word ―sangfroid‖, one could relate it with MS Dhoni, the ex-captain of the
Indian cricket team who was considered as captain cool and won India the World cup in 2011, the T20 world
cup in 2007 and The ICC campions trophy in 2013.He is the only international captain to do so. The word
―Sangfroid‖ means maintaining one‘s composure in trying or difficult circumstances and who did this better
than ―Mahi". In many finals, he took the Indian team through because of his composure and staying at the
crease till the end adapting himself to the conditions of the game. A true gem he was for Indian Cricket. That's
his story. The story of a sangfroid.

Similarly, stories can be made with other words as well. Here is a list of words to try out:

A. Preemptive
B. Audacity
C. Stardom
D. Consensus
E. Abysmal
A story with a word ―Adamant‖ could go like this:

I was a judge at one of the events in my college fest along with another colleague.

We were supposed to decide the results together. Although we came to a final list of candidates after a bit of
argument in which I recommended a name, but my colleague did not and finally we reached a consensus
wherein that candidate was omitted from the list yet afterwards I unanimously took the decision of including
that participant without informing my colleague. I was adamant and so did not think twice before taking such
a step.

This incident caused a rift between me, and my colleague and we could never be on positive terms with each
other from then.

75
Make similar stories to enthrall the audience. Messages are best conveyed when told in the form of a
story.

Email Writing
Why there is need to learn email writing?

As your career progresses the importance of communication skills increases. Email is how we spend most of
our time communicating, so it's your greatest opportunity. The email is often the first impression the receiver
gains of the sender. A poorly written email results in a poor first impression.

Although email is a valuable tool, it creates some challenges for writers. Miscommunication can easily occur
when people have different expectations about the messages that they send and receive. Email is used for
many different purposes, including contacting friends, communicating with professors and supervisors,
requesting information, and applying for jobs, internships, and scholarships. Depending on your purposes, the
messages you send will differ in their formality, intended audience, and desired outcomes.

When is email the appropriate form of communication to use?

Email is a good way to get your message across when:

● You need to get in touch with a person who is hard to reach via telephone, does not come to campus
regularly, or is not located in the same part of the country or world (for instance, someone who lives in
a different time zone).
● The information you want to share is not time sensitive. The act of sending an email is instantaneous,
but that does not mean the writer can expect an instantaneous response. For many people, keeping up
with their email correspondence is a part of their job, and they only do it during regular business
hours. Unless your reader has promised otherwise, assume that it may take a few days for them to
respond to your message.

76
● You need to send someone an electronic file, such as a document for a course, a spread sheet full of
data, or a rough draft of your paper.
● You need to distribute information to many people quickly (for example, a memo that needs to be sent
to the entire office staff).
● You need a written record of the communication. Saving important emails can be helpful if you need
to refer to what someone said in an earlier message, provide proof (for example, proof that you have
paid for a service or product), or review the content of an important meeting or memo.

When is email NOT an appropriate form of communication to use?

Email is not an effective means of communication when:

● Your message is long and complicated or requires additional discussion that would best be
accomplished face-to-face. For example, if you want feedback from your supervisor on your work or if
you are asking your professor a question that requires more than a yes/no answer or simple
explanation, you should schedule a meeting instead.
● The information is highly confidential. Email is NEVER private! Keep in mind that your message
could be forwarded on to other people without your knowledge. A backup copy of your email is
always stored on a server where it can be easily retrieved by interested parties, even when you have
deleted the message and think it is gone forever.
● Your message is emotionally charged, or the tone of the message could be easily misconstrued. If you
would hesitate to say something to someone‘s face, do not write it in an email.
Important components of an effective email:

Cc: and Bcc: (‗carbon copy‘ and ‗blind carbon copy‘)

Copying individuals on an email is a good way to send your message to the main recipient while also sending
someone else a copy at the same time. This can be useful if you want to convey the same exact message to
more than one person. In professional settings, copying someone else on an email can help get things done,
especially if the person receiving the copy is in a supervisory role. For example, copying your boss on an
email to a non-responsive co-worker might prompt the co-worker to respond. Be aware, however, that when
you send a message to more than one address using the Cc: field, both the original recipient and all the
recipients of the carbon copies can see all the addresses in the To: and Cc: fields. Each person who receives
the message will be able to see the addresses of everyone else who received it.

Blind copying emails to a group of people can be useful when you don‘t want everyone on the list to have
each other‘s addresses. The only recipient address that will be visible to all recipients is the one in the To:
field. If you don‘t want any of the recipients to see the email addresses in the list, you can put your own
address in the To: field and use Bcc: exclusively to address your message to others.

Subject Lines

Email subject lines are like newspaper headlines. They should convey the main point of your message or the
idea that you want the reader to take away. Therefore, be as specific as possible. One-word subjects such as
―Hi,‖ ―Question,‖ or ―FYI‖ are not informative and don‘t give the reader an idea of how important your
message is. If your message is time sensitive, you might want to include a date in your subject line, like
―Meeting on Thurs, Dec 2‖. Think about the subject lines on the email messages you receive. Which ones do
you think are most effective? Why?

77
Subject needs to be brief and concise. Remember to make it a practice. It must have the following needs:

1) Urgency

2) Curiosity

3) Offers

● Write the subject line after drafting your message


● Use action verbs so the reader knows what you want done
● Appeal to the reader‘s needs: ask yourself what will make the reader care about
● your email
● Keep your subject line under 50 characters or 6 to 8 words, so the whole line will
● show in the inbox preview
● Keep in mind that some smart phones show only 33 to 44 characters for the
● subject line
Salutation (Greetings and Signoffs)

Use greeting and sign-off. Don‘t just start with your text, and don‘t just stop at the end without a polite
signature. If you don‘t know the person well, you may be confused about how to address them (―What do I
call my TA/professor?‖) or how to sign off (Best? Sincerely?). Nonetheless, it is always better to make
efforts.

When in doubt, address someone more formally to avoid offending them. Some common ways to
address your reader are:

● Dear Professor Smith


● Hello, Ms. McMahon
● Hi, Mary Jane
If you don‘t know the name of the person you are addressing, or if the email addresses a diverse group,
try something generic, yet polite:

● To whom it may concern


● Dear members of the selection committee
● Hello, everyone
Your closing is extremely important because it lets the reader know who is contacting them. Always sign off
with your name at the end of your message. If you don‘t know the reader well, you might also consider
including your title and the organization you belong to; for example:

Mary Watkins

Senior Research Associate

Bain and Company

Joseph Smith

UNC-CH, Class of 2009

For your closing, something brief but friendly, or perhaps just your name, will do for most
correspondence:
78
● Thank you
● Best wishes
● See you tomorrow
● Regards
For a very formal message, such as a job application, use the kind of closing that you might see in a
business letter:

● Sincerely
● Respectfully yours
Opening Sentence

● Needs to be concise.
● Best to get straight to the point
● Include a maximum of four paragraphs and each paragraph should contain a single point.
● The first sentence can be a greeting if the situation allows it.
o I hope all is well with you.
o Thank you for your prompt response.
Starting Your Email

Replying to a previous email Giving brief updates

● ―Thanks for the information.‖ ● ―Just a quick note to tell you…‖


● ―Thanks for your phone call.‖ ● ―Just a quick note to let you know…‖

● ―Thanks for getting me the figures.‖ ● ―Just to update you on…‖

Referring to an attachment

● ―Take a look at the attached file.‖


● ―Have a quick look at the file I‘ve attached about…‖
● ―Thought you might find the attached interesting.‖

Middle Paragraphs

Content

● Give further useful details


● Try to discuss one idea per paragraph
● Or, if you‘re offering services or assistance, make sure it‘s clear what
● you can provide.
● Organize the rest of the information from most to least important
Language

● Keep your emails short.


● Keep a professional tone: avoid slang, exclamation marks, and smiley faces
● Write briefly and stick to the point: try to keep to 150 words or less
● Use short, everyday words instead of jargon and difficult words
● Keep sentences short

79
Middle Paragraph [sample]

At the feedback sessions on September 15 (www.badmanconference/sessions/feedback), our company will be


discussing ideas about using and hiding ―Really Big Lasers.‖ While we expect a lot of feedback from the
general audience, we would like your expertise on two points:

1. Your opinion of the project and the issues rose from the audience

2. Overseeing the quality of the refreshments provided by World Takeover

The above paragraph gives details about the conference and what the company wants James to do. The useful
details such as the date, topics and the conference website are all present in this email. James has everything
he needs to act.

Closing (Expressions for thanking)

● Thank you for your help. / time / assistance / support


● I really appreciate the help. / time / assistance / support you‘ve given me.
● Thank you once more for your help in this matter
Expressions with a future focus Closing

● I look forward to hearing from you soon / meeting you next Tuesday.
● I look forward to seeing you soon.
● I‘m looking forward to your reply.
● We hope that we may continue to rely on your valued custom.
● We look forward to a successful working relationship in the future.
● Please advise as necessary.
● I would appreciate your immediate attention to this matter.
Expressions for showing them you want to help

● If I can be of assistance, please do not hesitate to contact me.


● If you require any further information, feel free to contact me.
● If you require any further information, let me know.
● Please feel free to contact me if you need any further information.
● Please let me know if you have any questions.
● I hope the above is useful to you.
● Should you need any further information, please do not hesitate to contact me.
● Please contact me if there are any problems.
● Let me know if you need anything else
● Drop me a line if I can do anything else for you.
Signature

● Limit your signature to three or four lines of text


● Don‘t over think it. All you really need is:
Your Name

Title (optional) Company (linked to website)

Phone number

80
Don‘ts

1. Never substitute email for necessary face-to-face meetings

2. Don‘t assume that everyone reads email immediately

3. Never send an email message written in capital letters, LIKE THIS

4. Never forward jokes, spam, chain letters, or advertisements

5. Do not reply all who received an email unless it‘s relevant to them

6. Do not use email for any illegal or unethical purpose – goes without saying!

Sample Email

Dear Contact Name

Thank you for your offer of the position of Regional Manager of Product Development for the Witten
Company. I am impressed with the depth of knowledge of your development team and believe that my
experience will help to maximize the profitability of the department.

I would like to meet with you regarding the salary and benefits you have offered, before I make a final
decision. I feel that with the skills, experience, and contacts in the industry that I would bring to Witten,
further discussion of my compensation would be appropriate.

Thank you very much for your consideration.

Sincerely

Your Name

Email: [email protected]

Phone: 555-555-1234

Practice

1. As a former student, write an email to your professor, Mrs. Suzanne, thanking her for teaching and
guidance that contributed to your overall development. Sign the email as Sameer.

2. As a supplier, write an email to the manager of RD Wheel Company, Mr. Malhotra, intimating of their
payment that is due for the products delivered to them three months ago. Sign the email as Sameer.

3. You are a part of the corporate communication team in your company. The working time period is revised
as 8:30 am to 5:00 pm. Using the following phrases, write an email with a minimum of 70 words and a
maximum of 100 words to the employees in your company informing the same.

4. As a resident, write an email to the Municipal Commissioner of your city, Mr. Kumar, reporting nuisance
of a building under construction beside your place. Sign the email as Arvind.

81
5. As a student representative of your college, write an email to the Principal of Professional Engineering
College, Prof.Sanjib Chatterjee, inviting his institute to participate in the technical symposium being
organized in your college. Sign the email as Arun.

6. As your company is doing good business and expanding, your company is relocating its office to a new
address. Using the following phrases, write an email with a minimum of 70 words and a maximum of 100
words to your customer informing the change in address.

7. As a student representative of your department, write an email to your batch mates, suggesting a party for
Head of Department Prof. Mandy who is retiring next month. Sign the email as Shruti.

8. As an intern at XYZ consulting Pvt.Ltd, write an email to your internship Project Manager, Mr. Karunesh,
informing about the progress that you are making and some difficulties that you are encountering. Sign the
email as Max.

9. You are a project manager for a team of 20 resources. Write an email to your team, enquiring about the
irregularity in submitting their weekly timesheets and stressing the importance of the same.

10. As a supplier, write an email to the manager of M/S Big wheel Manufacturing Company, Mr. Chopra,
intimating of their payment that is due for the products delivered to them three months ago. Sign the email as
Ramesh

UNIT 4
ESSAY WRITING
What is Essay writing?

Essays are shorter pieces of writing that often require the student to hone a number of skills such as close
reading, analysis, comparison and contrast, persuasion, conciseness, clarity, and exposition. As is evidenced
by this list of attributes, there is much to be gained by the student who strives to succeed at essay writing.

The purpose of an essay is to encourage students to develop ideas and concepts in their writing with the
direction of little more than their own thoughts (it may be helpful to view the essay as the converse of a
research paper). Therefore, essays are (by nature) concise and require clarity in purpose and direction. This
means that there is no room for the student‘s thoughts to wander or stray from his or her purpose; the
writing must be deliberate and interesting.

The major types of essays:

1) Narrative Essays: Telling a Story

82
2) Descriptive Essays: Painting a Picture

3) Expository Essays: Just the Facts

4) Persuasive Essays: Convince Me

5) Argumentative Essays: Logic/ Argument

Narrative Essays: Telling a Story

In a narrative essay, the writer tells a story about a real-life experience. While telling a story may sound easy
to do, the narrative essay challenges students to think and write about themselves. When writing a narrative
essay, writers should try to involve the reader by making the story as vivid as possible. The fact that
narrative essays are usually written in the first person helps engage the reader. ―I‖ sentences give readers a
feeling of being part of the story. A well-crafted narrative essay will also build towards drawing a
conclusion or making a personal statement.

Descriptive Essays: Painting a Picture

A descriptive essay paints a picture with words. A writer might describe a person, place, object, or even
memory of special significance. However, this type of essay is not description for description‘s sake. The
descriptive essay strives to communicate a deeper meaning through the description. In a descriptive essay,
the writer should show, not tell, using colorful words and sensory details. The best descriptive essays appeal
to the reader‘s emotions, with a result that is highly evocative.

Expository Essays: Just the Facts

83
The expository essay is an informative piece of writing that presents a balanced analysis of a topic. In an
expository essay, the writer explains or defines a topic, using facts, statistics, and examples. Expository
writing encompasses a wide range of essay variations, such as the comparison and contrast essay, the cause-
and-effect essay, and the ―how to‖ or process essay.

Because expository essays are based on facts and not personal feelings, writers don‘t reveal their emotions
or write in the first person.

Persuasive Essays: Convince Me

While like an expository essay in its presentation of facts, the goal of the persuasive essay is to convince the
reader to accept the writer‘s point of view or recommendation. The writer must build a case using facts and
logic, as well as examples, expert opinion, and sound reasoning. The writer should present all sides of the
argument but must be able to communicate clearly and without equivocation because a certain position is
correct.

Argumentative Essays: Logic/ Argument

This is the type of essay where you prove that your opinion, theory or hypothesis about an issue is correct or
more truthful than those of others. In short, it is very similar to the persuasive essay (see above), but the
difference is that you are arguing for your opinion as opposed to others, rather than directly trying to
persuade someone to adopt your point of view.

⮚ Formal Writing Style


⮚ Complex – Longer sentences are likely to be more prevalent in formal writing. You need to be as thorough
as possible with your approach to each topic when you are using a formal style. Each main point needs to
be introduced, elaborated and concluded.
⮚ Objective – State main points confidently and offer full support arguments. A formal writing style shows a
limited range of emotions and avoids emotive punctuation such as exclamation points, ellipsis, etc., unless
they are being cited from another source.
⮚ Full Words – No contractions should be used to simplify words (in other words use "It is" rather than
"It's"). Abbreviations must be spelt out in full when first used, the only exceptions being when the
acronym is better known than the full name (BBC, ITV or NATO for example).
⮚ Third Person – Formal writing is not a personal writing style. The formal writer is disconnected from the
topic and does not use the first-person point of view (I or we) or second person (you).

84
Do‘s of Essay Writing:

Descriptive Elements:

1.Think about the dominant impression you want to give; don‘t attempt to describe every single thing.

2.Use imagery to put your reader right there with you.

3.Order events in time and space in such a way that the reader can imagine the location and time easily.

4.USE ADJECTIVES AND ADVERBS! Instead of: It was hot. In the afternoon, we rested on a log before
hiking further into the woods.

Try: Once the afternoon sun began to blaze down upon us, exhaustion set in, and we collapsed on a nearby
moss-covered log, groaning under the weight of its ancient branches. Our hike into the heart of the forest
would soon resume.

5. USE A THESAURUS! Instead of old, Try: elderly, aged, long in the tooth clean try: spotless, hygienic,
sterile

Instead ofgood,Try:marvelous, wonderful, incomparable, unrivaled

Instead of bad, Try: dreadful, horrific, appalling, ghastly.

Don‘ts of Essay Writing:

1. Using contractions (for example ‗I don‘t think‘ or ‗We can‘t say‘) instead of the full form (‗I do not
think‘ or ‗we cannot say.‘) ever use contractions in academic writing.
2. Writing too many words. The examiner is paid to mark on an ‗essay per hour basis,‘ and so will not read
the end of an essay if it exceeds the minimum word count by more than about 100 words. This means he
or she will not see the end of your argument, and your score will reduce considerably.
3. Using informal words (for example ‗a nice idea‘ or ‗a silly thing to do‘) instead of academic words (for
example ‗a positive idea‘ or ‗a regrettable thing to do.‘) Remember that academic vocabulary is different
from the language you would use in English when talking to friends.

85
1. Giving opinion in an IDEAS type Task. Check if the Task is asking for your opinion or not. The first
question you should ask yourself is ‗Is this an OPINION or an IDEAS Task?‘
2. Telling stories about your personal history, friends or family. The Task tells you to use ‗examples
from your own experience,‘ but this does not mean describing stories from your life or people you know!
It means describing examples of things in the world that you know about, have studied or have learned
about in the media.
3. Giving evidence which is too detailed or specific to a subject. You may be an expert in a particular
social or scientific field, but the examiner probably has a different specialty. You need to make your ideas
and examples accessible to a general reader. For example, if the Task topic is about money and you are an
accountant, do not use specialized accounting terms.
4. Being emotional or too dramatic when giving your opinion in an OPINION Task. You may feel
strongly about issues such as animals or crime, but academic writing must be unemotional. So, avoid
phrases such as ‗a disgusting idea‘ or ‗I detest this concept.‘ It is much better to say, ‗an unacceptable
idea‘ or ‗I disapprove of this concept,‘ which is more impersonal.
5. Academic in nature: An academic essay is one that is logically developed with the help of proper
reasoning. An essay should be well structured, concise and easy to comprehend.
6. Avoid ‗lazy‘ expressions (―and so on‖, ―etc‖).
7. Avoid SMS Language: We all are typing SMS messages, chatting on Skype and the like, and there is a
bunch of shorter ways to write longer words. We type ―u‖ instead of ―you‖, ―c‖ instead of ―see‖, ―IMHO‖
instead of ―in my humble opinion‖. None of these can appear in your IELTS exam, unless you are
specifically trying to mess up and get a lower score than you deserve. You need to write the full word and
spell it correctly, period.

The essay has the following standardized format:

1) Introduction

2) Elaboration

3) Conclusion

86
Something more about writing Introduction:

An attention-grabbing "hook" to introduce the topic. Example: write down mobile and its usage. instead
of directly starting from usage of mobile phones, start it with Technology and scenario.)

A thesis statement (demand of the essay, what should be written in body paragraphs and your opinion too
if required.) Something more about Thesis statement: A very basic thesis statement is one or two sentences
at the end of the first paragraph that tells the reader the main idea of your essay.

The Functions of a Thesis Statement:

• To announce the topic to the reader

• To reflect a judgment about the topic

• To provide the reader with a blueprint

• of what is to come in the paper

Sample:

As people live longer and longer, the idea of cloning human beings in order to provide spare parts is
becoming a reality. The idea horrifies most people, yet it is no longer mere science fiction.

Model Introduction for Human Cloning Essay

The cloning of animals has been occurring for several years now, and this has now opened the
possibility of cloning humans too. Although there are clear benefits to humankind of cloning to provide
spare body parts, I believe it raises several worrying ethical issues.

Body Paragraph 1: Topic Sentence:

87
Main Points:

88
1.
2.
3.
Body Paragraph 2: Topic Sentence:

Main Points:
1.
2.
3.
Sample Body Paragraphs: Due to breakthroughs in medical science and improved diets, people are living
much longer than in the past. This, though, has brought with it problems. As people age, their organs can
fail so they need replacing. If humans were cloned, their organs could then be used to replace those of sick
people. It is currently the case that there are often not enough organ donors around to fulfil this need, so
cloning humans would overcome the issue as there would then be a ready supply.

However, for good reasons, many people view this as a worrying development. Firstly, there are religious
arguments against it. It would involve creating another human and then eventually killing it in order to use
its organs, which it could be argued is murder. This is obviously a sin according to religious texts. Also,
dilemmas would arise over what rights these people have, as surely, they would be humans just like the rest
of us. Furthermore, if we can clone humans, it must be questioned where this cloning will end. Is it then
acceptable for people to start cloning relatives or family members who have died?

Conclusion: To conclude, I do not agree with this procedure due to the ethical issues and dilemmas it would
create. Cloning animals has been a positive development, but this is where it should end.

Another Sample: Popular events like the Football World Cup and other international sporting
occasions are essential in easing international tension and releasing patriotic emotions in a safe way.

Every four years, the whole world stops to watch international sporting events such as the Olympics and the
Football World Cup in which athletes show their best performance to make their country proud. These
sporting occasions have proved to be helpful in easing international tension in difficult times when powerful
leaders were trying to control the world‘s economy and other governments were fighting over the land.

89
The Olympic Games are one of the best examples which prove how sporting events can bring nations
together, at least temporarily. From the ancient History, when Greeks and Romans would interrupt battles to
participate in the games, to the more recent international disputes, when athletes from Palestine and Israel
would forget their differences, compete peacefully and even embrace each other after an event. Moreover,
these popular events have called the world‘s attention to the terrible consequences of wars; thus some
leaders have tried to reach agreements to end their disputes and live peacefully.

Similarly, international sporting events show benefits in some developing countries which live in a daily
internal civil war. For example, Brazil has a high rate of unemployment, lack of education, hunger, crime,
poverty and corruption which leads to an immense embarrassment of being Brazilian and a low self-esteem.
However, when the Football World Cup starts, the Brazilian squad, which is considered the best team in the
world, provokes an amazing feeling of pride in their country. Most people seem to forget all their problems
and even the criminal activity decreases. They paint roads with the national colors, wear the Brazilian team
shirts and buy national flags. Moreover, the competition brings families and neighbors together and even
rival gangs watch the games and celebrate peacefully.

In conclusion, popular sporting events play an important role in decreasing international tensions and
liberating patriotic feelings as history has shown.

Useful Phrases for essay writing:

⮚ Introducing the topic

● Currently / These days …

● These days, …

● Over the few past decades…

90
● Issues related to … are frequently discussed these days

91
● This essay will discuss … / will examine the arguments for and against / will examine the reasons why
people … / will consider the positive and negative impacts of this and discuss ways to…

● In my opinion, the potential dangers from this are greater than the benefits we receive.

● However, … is a contentious issue

● Although many people believe that…, others think that

● Although there are arguments on both sides, I strongly agree that…

● While early technologies certainly changed the lives of normal people, I believe that …

⮚ Stating an opinion (thesis statement), general phrases

● I believe that… / It is evident that / It has been claimed that… / It is widely believed that… / It is
certainly true to say that… / It is accepted that… / Many people consider that…

● There is no doubt that…

● Not only … but it could also…

● First and foremost, …

● The principal reason that people … is because

● Something has increasingly … / people are increasingly using

● Something takes advantage of (natural light)

● There are many advantages (benefits)to (using the Internet)

● Something may be endangered

● Something aims to (teach young people moral values)

● Government should introduce …

● Something is at risk of …

● … a responsibility towards (the older people)

⮚ Giving reasons

● One reason for this is that…

● This can be explained by…

● This can be attributed to the fact that…

92
● It should also be stated that…

93
⮚ Refuting opinions

● I would dispute this, however.

● However, this is not necessarily the case.

● There are some faults with this reasoning, however.

● On the other hand, …

⮚ Concluding and summarizing

● In conclusion,

● To conclude…

● To sum up…

● Overall, … / On the whole, …

● I strongly believe that…

● In conclusion, it is clear that

● The advantages (benefits)of … outweigh the disadvantages (drawbacks).

⮚ Linking phrases

Nevertheless, however, although, in addition, therefore, furthermore, moreover, also, despite this,
finally, in fact, indeed, consequently, eventually, thereafter … (see speaking brief file)

Templates for Essay:

Introduction

Trend description

- It is certainly true that / There is no doubt that X is increasingly being used to Y nowadays.

- It is certainly true that [populations in developed countries] are increasingly [overweight nowadays].

- Nowadays more and more [specialists] are Z to Y

- Over the past few decades, Z has improved dramatically, [allowing Y to X] [introducing innovative X],
which has changed many aspects of our everyday life

- Over the decades/centuries, [generations of] scientists have been struggling to find answers to the most
complicated questions about X. However, some of them have not been resolved yet. OR been struggling
to understand the nature of X

94
- Nowadays many people increasingly prefer X rather than Y

- Nowadays parents/people can choose whether to Z or X

95
Problem

- It is certainly true that people are increasingly [producing more garbage], which has had a significant
negative effect on the environment.

- While many people believe that the situation may be improved by Y, I do not consider this as an effective
measure and think that other approaches can be more useful.

People think

- It is certainly true that issues/questions related to Z, [such as Y], are frequently discussed these
days/nowadays since/due to the fact that... In particular, many people complain that…

- It is often argued that …

- It is currently a contentious argument over whether…

- It is certainly true that X is a contentious topic for many adults/people.

- It is certainly true that the attitude towards [higher education] is different all over the world.

- It is certainly true that X is a contentious topic which raises many ethical questions.

- There are different opinions in regard to the question of [who should teach X about Z]

- It is certainly true to say that such phenomenon as X is frequently discussed nowadays.

- This still remains a controversial topic for many people.

- These factors have led to a phenomenon known as Z, which is criticized by many people.

- Some people consider this phenomenon as negative, whereas others believe that they benefit from Z /
whereas I believe that society can benefit from Z.

- A commonly held belief is that...

Body

Body paragraph opinion essay starting

- The main reason why I am convinced that Z is because Y.

- Another reason why I support the idea that Y is that X.

96
Estimation of good / bad

- Z can be beneficial in many ways / have a positive (favourable) effect

- Z increases the quality of everyday life.

- Z can have an adverse effect on [youths] / can affect [youths] adversely.

- These factors can have a negative effect on Z

- X can pose a threat to [healthy relationships]

- The benefits / the potential positive effects [of X] are outweighed by a number of drawbacks.

- There are several obvious shortcomings related to X.

- The downside (singular!) of ... is that

Agree/disagree

- Somebody subscribes to the view that...

- Somebody concurs that (agree without hesitation)

Arguing

- Although Y, this is not necessarily the case nowadays.

- Another significant reason why X is …

- Something can produce noticeable increase in the quality of life.

- Many factors may be more important than X

- Thus, it has subsequently become apparent that …

- From this it becomes evident that …

- Thus, it is clear that …

Problems

- [The increase in X] has been caused by a variety of factors

- Something stems from (there is a direct link)

- The effects of this are extremely serious.

- Y may not be the best way to tackle the issue for several reasons.

- Alternatively, a number of comprehensive measures can be taken to solve X problems.

- The best way to solve the problem would be to …


Page | 97
- By doing this, they would [feel better generally]

- It is important that ...

Info and data support

- Z is a key part of modern business.

- From my knowledge, some recent stories in the media support this belief/point of view.

- Some recent stories in the media support the belief that … (stories – better not use in opinion essay)

- This is yet another example of how ...

- This is/was true of somebody, who ...

Conclusion

Restating estimation of good / bad

- There are a number of advantages and disadvantages regarding both approaches.

- The benefits of Z are outweighed by several disadvantages / by several drawbacks.

- This may have either a positive or a negative effect, depending on X

Stating approaches

- I think that Z should supplement Y with the use/means of N.

- Otherwise, potential dangers may overweight possible benefits.

- People should be aware of risks related to X and [use modern means of communication] wisely.

- [Every adult] should think thoroughly/deliberately [prior to encouraging their children to do so].

- There is no reason why [x and y] cannot exist together.

Problems

- I am convinced that problems related to X should be tackled not only by Y, but by the implementation of
other measures as well. This can include…

- In conclusion, X is clearly alarming, and Z should tackle the issue by the implementation of a set of
comprehensive measures.

- As a result, … which will have a significant positive effect on [the environment].

98
- I am convinced that such a comprehensive approach is the best way to eliminate the root cause of many
[social] problems.

99
- Unless these actions are taken, the devastating effect on [the environment] may be irreversible.

EXERCISE: Identify the type of essay in each statement and write 300-350 words.

1.According to some people, elderly drivers should be required to reapply for their driving licenses because
with age comes diminished vision, hearing, and reaction time. How do you feel about this issue? Explain
what you think should be done and why.

2.Have you ever travelled to a place that you found very meaningful and rewarding? Write an essay that
persuades others to visit this important place.

3.If it were up to you to choose one item from the twenty-first century to place in a time capsule for future
generations, what would you choose? Use specific reasons and examples to support your choice, explaining
both the item‘s significance and the reasons why it embodies the culture of the early twenty first century.

4.Explain the causes and effects of not voting in elections.

5.Many people spend a great deal of time with animals. Write about the relationships that people have with
animals.

6.People often say, ―Don‘t judge a book by its cover.‖ Describe a time when you misjudged someone based
on his or her appearance or when someone misjudged you.

7. Carpooling, recycling, and planting trees are all activities that are good for the environment. Write an
Essay convincing readers to actively participate in one of these activities.

Practice

Write an essay in 300-350 words

● Freedom of speech: To what extent can it be exercised?


● Discuss the pros and cons of Agnipath scheme.
● Can India become a superpower? What are the challenges it faces on this path?
● How can we make the world a better place to live in?
● Religious bigotry: Can we come out of it?
● UGC has come up with a proposal to have 30% of the course of any specialization imparted through
the online mode. Weigh its pros and cons.

● In the developed world, technological progress is increasing. What problems will this cause for
individuals and society? Suggest some measures that could be taken to reduce the impact of radical
technological advancements.
● In today‘s very competitive world, a worker must possess multiple skills to succeed. Among the skills
that a worker should possess, which skill do you think is more important, social skills or good
qualifications? Explain the reasons and provide specific examples to support your answer.
● Do you think businesses should hire employees who will spend their entire lives working for the
company? Explain why you agree or disagree. Use specific reasons and details to support your answer.

100
● Some people think women should be given equal chances to work and excel in their careers. Others
believe that a woman‘s role should be limited to taking care of the house and children. Which opinion
do you agree with and why? Include specific details and examples to support your choice.

101
Exercise

● Corruption is the single biggest challenge that the country faces today. To what extent do you agree
with this statement?
● Rising Inequality in the country.
● A literate mind vs a well-educated mind. Compare and contrast using suitable examples.
● Can reservation system be removed. Discuss concrete ways to do so.
● Climatic Change and its impact on the planet Earth.
● One who controls the media controls the world. Do you agree with this statement.
● Should attendance be compulsory in schools and colleges?
● Higher Education vs Skill Acquisition.
● Global Terrorism- The fight against terrorism
● Cashless Economy: Is it achievable?

102
Reading Comprehension
Reading comprehension is the act of understanding what you are reading. Reading comprehension is an
intentional, active, interactive process that occurs before, during and after a person reads a particular piece of
writing. Reading comprehension is one of the pillars of the act of reading. There are two elements that make
up the process of reading comprehension: vocabulary knowledge and text comprehension. In order to
understand a text, the reader must be able to comprehend the vocabulary used in the piece of writing. If the
individual words don‘t make the sense, then the overall story will not either.

General Strategies for Reading Comprehension

There are six main types of comprehension strategies (Harvey and Goudvis; 2000):

1. Make Connections—Readers connect the topic or information to what they already know about
themselves, about other texts, and about the world.

103
1. Ask Questions—Readers ask themselves questions about the text, their reactions to it, and the author's
purpose for writing it.

2. Visualize—Readers make the printed word real and concrete by creating a ―movie‖ of the text in their
minds.

3. Determine Text Importance—Readers (a) distinguish between what's essential versus what's interesting,
(b) distinguish between fact and opinion, (c) determine cause-and-effect relationships, (d) compare and
contrast ideas or information, (e) discern themes, opinions, or perspectives, (f) pinpoint problems and
solutions, (g) name steps in a process, (h) locate information that answers specific questions, or (i)
summarize.

4. Make Inferences—Readers merge text clues with their prior knowledge and determine answers to
questions that lead to conclusions about underlying themes or ideas.

5. Synthesize—Readers combine new information with existing knowledge to form original ideas, new lines
of thinking, or new creations.

8 Types of Reading Comprehension Questions

1. Universal or Main Idea Questions – This question-type asks about the big picture, the passage as a
whole.

● ―Which of the following best summarizes the passage?‖

● ―What is the author attempting to illustrate through this passage?‖

● ―What is the thematic highlight of this passage?‖

● Which of the following best states the central idea of the passage?
● The gist of the passage is:
● Which of the following is the principal topic of the passage?

● ―Which of the following best describes one of the main ideas ____?‖

2. Specific –This type asks about details from the passage. The correct answer is often a paraphrase of
something directly stated in the passage.

● ―Which _____ has not been cited as ______?‖

● ―According to the author, what is ______?‖

● ―By a _______, the author means…‖

● ―According to the passage, _______?‖

● ―Which factor has not been cited _______?‖

104
3. Vocab-in-Context – This is a type of Specific question which asks about the use of a particular word or
phrase.

● ―In the passage, the phrase _____ refers to‖

● ―In the sentence, _______, what is the meaning of ‗___‘?‖

4. Function – This type of question asks about the logical structure of a passage.

● ―The author cites ______, in order to‖

● ―The _____ in the passage has been used by the author to‖

● ―Which of the following best describes the reason the author ____?‖

5. Inference – Inference questions require you to understand what is implied by but not necessarily stated
in the passage. The correct answer may rely on subtle phrases from the passage and be hard to find/less
obvious than Specific questions.

● ―The passage uses _____ to imply that ___‖

● ―Which of the following cannot be inferred from the passage?‖

● ―What does the author mean by _____?‖

● ―What can be inferred when the author states____?‖

● The sentence, ‗______‘, implies that‖

6. Application – This is a slightly more specific type of inference question, where you‘re asking to choose
an answer which mimics a process or exemplifies a situation described in the passage.

● Which of the following could be used to replace_______?

● ―A _____, as conceptualized in the passage, can best be described as____‖

● ―Which of the following best illustrates the situation ____‖

7. Tone – This question type is also a specific type of inference, requiring you to infer the author‘s point
of view and position on certain statements.

● ―The author of the passage is most likely to agree with ____?‖

● ―Which of the following views does the author most likely support__?‖

8. Context outside passage -Like inference questions, applying information questions deal with topics
that are not mentioned explicitly in the passage. This type of question asks you to take the information
given in the passage and apply it logically to a context outside of the passage. An applying information
question could look like:

● Which of the following situations is most similar to the situation described in the paragraph?

105
● Identify a hypothetical situation that is comparable to a situation presented in the passage?

● Select an example that is like an example provided in the passage?

Main idea 1

There are many reasons why the death penalty should have minimal place in any society, not least
because it violates the fundamental right to life. The argument that it may deter violent crime is
countered by the observations that murder rates declined in ten out of eleven countries which had
abolished capital punishment in recent years. The most egregious aspect of the death penalty is the
widespread evidence of miscarriage of justice which occurs even in the most robust judicial systems,
leading to the real threat of an irreversible punishment being inflicted on an innocent person.

What is the main idea of the passage?

A. Death penalty should not be abolished.

B. Death penalty should be reduced to minimum.

C. It is debatable whether the death penalty should be abolished or not.

D. None of these

Main Idea 2

It is inevitable that the ban on polythene bags by the government will encounter teething troubles, such
as resistance from a section of the trader community as well as the industry. The end-users will also feel
the pinch if the substitutes (paper/cloth bags) are not easily available or lack affordability and
durability. Several states have imposed a complete or partial ban on single-use plastic over the years,
including Karnataka, Maharashtra, Tamil Nadu and Himachal Pradesh. The implementation has been
patchy in general, with the authorities often adopting a lax attitude and things eventually returning to
business as usual.

What is the main idea of the passage?

A. The recent ban on polythene will prove to be an all-out success.


B. The ban on polythene will face resistance from a certain section of the society including traders and
industrialists.
C. The ban on polythene is a good step towards clean and green India.
D. Ban on polyethene in the past has not been that successful in some states, so the recent ban in the
entire country is also going to face some challenges.

106
Main Idea 3

The attitudes of Americans toward gambling are amazingly contradictory. You may find, for
example, that horse racing is legal in your state, but that you cannot legally play poker for money on
your front porch; bookies may be prosecuted by state law, but they are supposed to purchase a
federal license nonetheless; one church condemns gambling, while another raises money by
sponsoring Bingo games. Gambling laws are inconsistent from state to state or even from town to
town and are very difficult to enforce.

A. Americans have negative attitudes toward gambling.


B. Gambling laws are difficult to enforce
C. Gambling laws are inconsistent from community to community.
D. Churches do not have uniform ideas about gambling.

Specific - 1

Naval architects never claim that a ship is unsinkable, but the sinking of the passenger-and-car ferry Estonia
in the Baltic surely should never have happened. It was well designed and carefully maintained. It carried the
proper number of lifeboats. It had been thoroughly inspected the day of its fatal voyage. Yet hours later, the
Estonia rolled over and sank in a cold, stormy night. It went down so quickly that most of those on board,
caught in their dark, flooding cabins, had no chance to save themselves: Of those who managed to scramble
overboard, only 139 survived. The rest died of hypothermia before the rescuers could pluck them from the
cold sea. The final death toll amounted to 912 souls. However, there were an unpleasant number of questions
about why the Estonia sank and why so many survivors were men in the prime of life, while most of the dead
were women, children and the elderly.

107
According to the passage, when the Estonia sank, ----.

A) there were only 139 passengers on board

B) few of the passengers were asleep

C) there were enough lifeboats for the number of people on board

D) faster reaction by the crew could have increased the Estonia's chances of survival

E) all the passengers had already moved out into the open decks

Vocab in context - 1

Her paintings are now very famous and, although the dark colours may not be attractive to some people,
they evoke the beauty and mystery of the deep woods and the skill of a great artist.

The word "Evoke "in the passage means:

a) reason
b) discover
c) make you feel
d) disclose

Function – 1

Seeking a competitive advantage, some professional service firms (for example, firms providing advertising,
accounting, or health care services) have considered offering unconditional guarantees of satisfaction. Such
guarantees specify what clients can expect and what the firm will do if it fails to fulfill these expectations.
Particularly with first-time clients, an unconditional guarantee can be an effective marketing tool if the client
is very cautious, the firm‘s fees are high, the negative consequences of bad service are grave, or business is
difficult to obtain through referrals and word-of-mouth.

However, an unconditional guarantee can sometimes hinder marketing efforts. With its implication that
failure is possible, the guarantee may, paradoxically, cause clients to doubt the service firm‘s ability to
deliver the promised level of service. It may conflict with a firm‘s desire to appear sophisticated, or may even
suggest that a firm is begging for business. In legal and health care services, it may mislead clients by
suggesting that lawsuits or medical procedures will have guaranteed outcomes. Indeed, professional service
firms with outstanding reputations and performance to match have little to gain from offering unconditional
guarantees. And any firm that implements an unconditional guarantee without undertaking a commensurate
commitment to quality of service is merely employing a potentially costly marketing gimmick.

Which of the following is cited in the passage as a goal of some professional service firms in offering
unconditional guarantees of satisfaction?

A. A limit on the firm's liability

108
B. Successful competition against other firms

C. Ability to justify fee increases

D. Attainment of an outstanding reputation in a field

Inference – 1

Jessie Street is sometimes called the Australian Eleanor Roosevelt. Like Roosevelt, Street lived a life of
privilege, but she devoted her efforts to working for the rights of the disenfranchised, including workers,
women, refugees, and Aborigines. In addition, she gained international fame when she was the only woman
on the Australian delegation to the conference that founded the United Nations, just as Eleanor Roosevelt
was for the United States.

Which of the following inferences may be drawn from the information presented in the passage?

a. Eleanor Roosevelt and Jessie Street worked together to include women in the United Nations Charter.

b. Usually, people who live lives of privilege do not spend much time participating in political activities.

c. Discrimination in Australia is much worse than it ever was in the United States.

d. At the time of the formation of the United Nations, few women were involved in international affairs.

Application – 1

The piranha is a much-maligned fish. Most people think that this is a deadly creature that swarms through
rivers and creeks of the Amazon rainforest looking for victims to tear apart. And woe betide anyone unlucky
enough to be in the same water as a shoal of piranhas. It takes only a few minutes for the vicious piranhas to
reduce someone to a mere skeleton. The truth is that the piranha is really a much more nuanced animal than
the mindless killer depicted in the media. In fact, piranhas are a group made up of approximately twelve
different species. Each piranha species occupies its own ecological niche. One type of piranha takes chunks
out of the fins of other fish. Another type eats fruit falling from trees into the river. Each piranha species
plays a unique role in the ecology of the rainforest floodplains. So, what should you do next time you hear
someone talking about the ―deadly piranha‖? You can remind them that the piranha is not always the
notorious killer fish that the tough, muscular heroes of popular nature television shows would have us
believe.

In paragraph 1, the author uses hyperbole—characterized by the use of exaggeration for effect—to
describe the piranha. The author most likely uses hyperbole to

A. frame an argument that is supported in a later paragraph

B. create ambiguity so the reader cannot be sure which position the writer supports

C. juxtapose the myth of the piranha with the truth about the fish

D. evoke vivid images of nature television shows in the reader‘s mind

109
Tone :

Some of the common tones used by author are listed below:

✔ Critical: denotes negative or fault-finding attitude of the author. Sometimes, the word critical is also
used to denote deep analysis of the issue with a neutral outlook.
✔ Laudatory/Eulogistic: To shower high praise on something or somebody.
✔ Cynical: A higher degree of pessimism coupled with a sense of skepticism and helplessness.
✔ Satirical: To use humour as a tool for healthy criticism.
✔ Sarcastic: To use the words opposite to what you mean to taunt or make fun of somebody.
✔ Didactic: When the author tries to teach or instruct through his writing.
✔ Nostalgic: Conveys a sense of longing for the past.

Solutions:

Main Idea 1 Main Idea 2 - D Main Idea 3 - C Specific 1 - C


-B

Vocab in Function 1 - B Inference 1 - D Application 1 – C


Context 1 –
C

TIPS

110
● Don't rush yourself. It is natural that you will start a little slowly and then build up speed as you gain
familiarity with the passage.

111
● Read all the questions first (if the exam format permits this). This will help you zero in on the
relevant portions when you start reading the passage.

● Read actively. This means, try to anticipate the next sentence. Reading this way will help you engage
with the passage more closely.

● Make notes to capture the essence of each paragraph within the passage. The first sentence of the
paragraph usually conveys the main idea or theme of the paragraph.

● Don't try to memorize anything. Memorizing consumes time and is not very useful.

● Get the overview. After you have read the passage, ask yourself the following questions: (1) what is
the passage trying to say? (2) How does each paragraph contribute to the broad message of the
passage?

Practice

PASSAGE 1.

It is difficult to reconcile the ideas of different schools of thought on the question of education. Some people
maintain that pupils of school should concentrate on a narrow range of subjects which will benefit them
directly in their subsequent careers. Others contend that they should study a wide range of subjects so that
they not only have the specialized knowledge necessary for their chosen careers but also sound general
knowledge about the world they will have to work and live in. Supporters of the first theory state that the
greatest contributions to civilization are made by those who are expert in their trade or profession. Those on
the other side say that, unless they have a broad general education, the experts will be too narrow in their
outlook to have sympathy with their fellows or a proper sense of responsibility towards humanity as a whole.

(1) Supporters of the first theory would not agree with

A. experts have contributed most to the progress in the modern world

B. students should study a few subjects that will help them in their profession

C. people with broad general education have not contributed much to civilization

D. students should not undertake any specialized work

(2) According to the second school of thought education will not be very effective, if students

A. do not have a wide general education

B. have inadequate knowledge of their own work

C. ignore the study of fine arts

D. concentrate on only a few subjects

PASSAGE 2

112
While delivering the Waheeduddin Khan Memorial Lecture on ‗Beyond India @75: Growth, Inclusion and
Sustainability‘ at the Centre for Economic and Social Studies (CESS) on Thursday, Prof. Dev said that there
are issues like geopolitical challenges with a new bipolar world — USA/China — emerging in the recent past
with old bipolar world — USA/Russia, and international trade and protection, and technologies like
automation, robotics, 3D printing, digitization and Artificial Intelligence and increasing inequalities.

Explaining various issues that would impact the economic growth of India, Prof. Dev said that the
government has mentioned that India would achieve $ 5 trillion economy by 2024 but it would take another
four to five years to reach that. Informing that export of goods has completely stagnated with annual growth
rate of almost 0% during 2012-19 while the growth rate of service exports declined noticeably to 5.9%. ―India
accounts for less than 2% of the world export manufacturers while the share of China stands at 13%,‖ he
commented.

Referring to credit in the national economy, he said that it was only 50% of GDP compared to 100% in many
other countries. He stressed the need to keep credit flowing to all categories of economic agents – firms and
households. On agriculture, he said that the current policies are still based on the ‗deficit‘ mindset of the
1960s and the procurement, subsidies and water policies are biased towards rice and wheat.

―Diversification of cropping pattern towards millets, pulses, oil seeds and horticulture is needed for more
equal distribution of water, sustainable and climate resilient agriculture,‖ said Prof. Dev, adding that
agriculture should change towards high value production, better remunerative prices and farm income.

Q1. Acc. to the paragraph, India faces challenges in all of these fields except:

A. Geopolitics

B. Inequalities

C. Diversification of cropping pattern

D. Gun Control

Q2. The difference in the world exports between China and India is:

A. 13%

B. 2%

C. 11%

D. 9%

Q3. What do you understand about the ―deficit‖ mindset of the 1960s?

A. There was a shortage of food grains at that time.

B. There was an abundance of food grains at that time.

113
C. There was neither shortage nor abundance of food grains.

D. None of these

Q4. Diversification of cropping pattern is required towards:

A. Horticulture

B. Millets & Pulses

C. Oil seeds

D. All of the above

Q5. What was the growth rate of Indian exports from 2012 to 2019?

A.1%

B.2%

C.0%

D.4%

114
115
PASSAGE 3

But man is not destined to vanish. He can be killed, but he cannot be destroyed, because his soul is deathless,
and his spirit is irrepressible. Therefore, though the situation seems dark in the context of the confrontation
between the superpowers, the silver lining is provided by amazing phenomenon that the very nations which
have spent incalculable resources and energy to produce deadly weapons are desperately trying to find out
how they might never be used. They threaten each other, intimidate each other and go to the brink, but before
the total hour arrives, they withdraw from the brink.

The main point from the author's view is that

A. Man's soul and spirit cannot be destroyed by superpowers.


B. Man's destiny is not fully clear or visible.
C. Man's soul and spirit are immortal.
D. Man's safety is assured by the delicate balance of power in terms of nuclear weapons.
E. Human society will survive despite the serious threat of total annihilation.

The phrase 'Go to the brink' in the passage means

A. Retreating from extreme danger.


B. Declare war on each other.
C. Advancing to the stage of war but not engaging in it.
D. Negotiate for peace.
E. Commit suicide.

In the author's opinion

A. Huge stockpiles of destructive weapons have so far saved mankind from a catastrophe.
B. Superpowers have at last realized the need for abandoning the production of lethal weapons.
C. Mankind is heading towards complete destruction.
D. Nations in possession of huge stockpiles of lethal weapons are trying hard to avoid actual conflict.
E. There is a silver lining over the production of deadly weapons.

'Irrepressible' in the second line means

A. incompatible
B. strong
C. oppressive
D. unrestrainable
E. inspirited

116
A suitable title for the above passage is

A. Destruction of mankind is inevitable.


B. Man's desire to survive inhibits use of deadly weapons.
C. Mounting cost of modern weapons.
D. Threats and intimidation between superpowers.
E. Cowardly retreat by man

The silver lining is provided by amazing phenomenon. What is the synonym for the italicized word?

A. Despair
B. Bright side
C. Disposition
D. Hopelessness

The word confrontation in the passage means

A. Discord resulting from a clash of ideas or opinions


B. A direction opposite to another
C. An electrical device that resists the flow of electrical current
D. The state of not being susceptible

What do we mean by intimidate?

A. Threaten
B. Pacify
C. Love
D. Call

PASSAGE 4

The Food and Drug Administration has formulated certain severe restrictions regarding the use of antibiotics,
which are used to promote the health and growth of meat animals. Though the different types of medicines
mixed with the fodder of the animals kills many microorganisms, it also encourages the appearance of
bacterial strains, which are resistant to anti-infective drugs.

It has already been observed that penicillin and the tetracyclines are not as effective therapeutically as they
once used to be. This resistance to drugs is chiefly caused due to tiny circlets of genes, called plasmids, which
are transferable between different species of bacteria. These plasmids are also one of the two kinds of vehicles
on which molecular biologists depend on while performing gene transplant experiments. Existing guidelines

117
also forbid the use of plasmids, which bear genes for resistance to antibiotics, in the laboratories. Though
congressional debate goes on as to whether these restrictions need to be toughened with reference to scientists
in their laboratories, almost no congressional attention is being paid to an ill-advised agricultural practice,
which produces deleterious effects.

In the present passage, the author's primary concern is with:

A. The discovery of methods, which eliminate harmful microorganisms without generating drug-resistant
bacteria.
B. Attempting an explanation of the reasons for congressional inaction about the regulation of gene
transplant experiments.
C. Portraying a problematic agricultural practice and its serious genetic consequences
D. The verification of the therapeutic ineffectiveness of anti-infective drugs
E. Evaluation of the recently proposed restrictions, which are intended to promote the growth of meat
animals.

As inferred from the above passage, the mutual transfer of plasmids between different bacteria can
result in which of the following?

A. Microorganisms, which have an in-built resistance to drugs


B. Therapeutically useful circlets of genes
C. Penicillin like anti-infective drugs
D. Viruses used by molecular biologists
E. Carriers for performing gene transplant experiments.

According to the above passage the author believes that those who favor the stiffening of restrictions on
gene transplant research should logically also.

A. Approve and aid experiments with any plasmids except those, which bear genes for antibiotic
resistance.
B. Inquire regarding the addition of anti-infective drugs to livestock feeds
C. Oppose the using of penicillin and tetracyclines in order to kill microorganisms
D. Agree to the development of meatier livestock using antibiotics
E. Approve of congressional debate and discussion regarding science and health issues.

The attitude the author has with reference to the development of bacterial strains that render antibiotic
drugs in effective can best be described as

A. indifferent
B. perplexed
C. pretentious
D. insincere

118
E. apprehensive

The Food and Drug Administration has formulated certain severe restrictions

A. Stringent
B. Dangerous
C. Terrible
D. Wicked

Passage 5

Visual recognition involves storing and retrieving memories. Neural activity, triggered by the eye, forms an
image in the brains memory system that constitutes an internal representation of the viewed object. When an
object is encountered again, it is matched with its internal representation and thereby recognized. Controversy
surrounds the question of whether recognition is a parallel, one-step process or a serial, step-by-step one.
Psychologists of the Gestalt school maintain that object are recognized as wholes in a parallel procedure: the
internal representation is matched with the retinal image in a single operation. Other psychologists have
proposed that internal representation features are matched serially with an object's features. Although some
experiments show that, as an object become familiar, its internal representation becomes more familiar, its
internal representation becomes more holistic and the recognition process correspondingly more parallel, the
weight of evidence seems to support the serial hypothesis, at least for objects that are not notably simple and
familiar.

It can be inferred from the passage that the matching process in visual recognition is

A. Not a natural activity.


B. Not possible when an object is viewed for the very first time.
C. Not possible if a feature of a familiar object is changed in same way.
D. Only possible when a retinal image is received in the brain as a unitary whole.
E. Now fully understood as a combination of the serial and parallel process.

In terms of its tone and form, the passage can best be characterized as

A. A biased exposition
B. A speculative study
C. A dispassionate presentation
D. An indignant denial
E. A dogmatic explanation.

The author is primarily concerned with

A. Explaining how the brain receives images


B. Examining the evidence supporting the serial recognition hypothesis

119
C. Discussing visual recognition and some hypotheses proposed to explain it.
D. Reporting on recent experiments dealing with memory systems and their relationship to neural
activity.

According to the passage, Gestalt psychologists make which of the following suppositions about visual
recognition?
I A retinal image is in the same form as its internal representation
II An object is recognized without any need for analysis into component parts.
III The matching of an object with its internal representation occurs in only one step

II only

III only

I and III only

II and III only

I, II and III

120
PASSAGE6

Any modern musical performance is almost impossible to countenance without the presence of an electric
guitar. Most of the time it is a solid-body electric guitar, and while they seem ubiquitous and obvious now,
that was not always the case. First invented in the early 1930s, the first electric guitar simply amplified
existing guitars. No one thought of it as a new instrument, but merely a way to put a microphone inside of the
guitar. Using electronic pickups that went straight to an amplifier, the sound of the guitar could be broadcast
over loud jazz bands with drums and horns. At the time, most everyone believed an electric guitar still had to
look like an acoustic guitar, and all models featured a hollow body acoustic shape that would resonate with
the sound of the guitar strings. In all actuality, the only necessity for an electric guitar is an electric pickup to
capture their small vibrations. An electric guitar does not, and never did, need a space to resonate the sound of
the strings. Instead, it could be a simple block, with the fretboard, strings, and a pickup attached to a piece of
lumber. This method is exactly what the famous guitar player and maker Les Paul did with his ―Log,‖ but Les
Paul's ―Log‖ revealed some of the biases against a solid-body guitar. While the guitar was just one solid piece
of wood, Paul would attach two wings to it that made the guitar look like a hollow body.

121
Despite Les Paul‘s innovations, few manufacturers made a marketable solid-body guitar. Rickenbacker and
Bigsby were both companies that made limited productions of solid-body electric guitars. Leo Fender was the
first luthier to make a popular, mass-market electric solid-body guitar. Leo Fender started his career by
working on radios and other small electronic devices but developed an interest in building guitars.
Immediately after World War II, big bands were considered antiquated, and small honky-tonk and boogie-
woogie combos wanted cheaper, sturdier, and better intonated guitars, that they could play faster and louder.
Leo Fender obliged with his Esquire guitar. Looking completely unlike any guitar made before, and being
extremely thin, with no resonating panels, Fender‘s guitar was revolutionary. While Fender continued to
tweak it through the years, one thing remains the same: the general shape of the guitar. Renamed first the
Broadcaster, then the more famous Telecaster, the silhouette of Fender‘s Esquire is still a popular choice
among musicians today.

Q. It can be inferred from the passage that the Bigsby and Rickenbacker companies ________.

A. were not highly innovative guitar manufacturers

B. took many of their ideas from Leo Fender's innovations

C. helped inspire the creation of Les Paul's "Log" guitar

D. never had any success in manufacturing guitars

F. were never as successful as Fender in marketing their guitars

Explanation:

The Bigsby and Rickenbacker companies are only mentioned in one sentence in the entire passage, where it is
noted, they too made a small run of electric solid-body guitars. From this, it can be inferred that they had
some success, but never achieved anywhere near the level of Leo Fender's company.

PASSAGE 7

When winding an old clock, it is important not to overwind it. Overwinding occurs when the mainspring is
almost fully wound, but the operator continues to turn the winding key. This causes the main spring to coil too
tightly and might even break it.

Q. what is the main idea of this paragraph

a. clocks have changed over the years.

b. old-fashioned clocks become fragile with age.

c. old-fashioned clocks were operated by an internal spring.

d. overwinding clocks used to be a common mistake.

e. time flies when you're having fun.

PASSAGE 8

122
Identify the tone of the passage

All registered cyclists have passed an examination covering traffic rules and safety. Those interested in the
new Bikeshare Program must register their bicycles and provide a major credit card to ensure payment. Call
555-1212 for more information.

123
(A) argumentative
(B) sentimental
(C) idealistic
(D) informative
(E) caustic
PASSAGE 9
Identify the tone of the passage

On hot summer nights his family drove to the local ice cream stand, where, for just a few minutes, nothing
mattered but the cold, sweet treat. No dessert, before or since, was as delicious as a double-dip cone eaten
while he balanced on a log behind the parking lot.

(A) nostalgic

(B) biting

(C) irreverent

(D) cynical

(E) satirical

PASSAGE 10

Although European decisions during the 16th and 17th centuries to explore, trade with, and colonize large
portions of the world brought tremendous economic wealth and vast geographic influence, the enormous
success of European maritime ventures during the age of exploration also engendered a litany of unintended
consequences for most of the nations with which Europe interacted. Due to their incredible military force,
religious zeal, and uncompromising goal of profit, Europeans often imposed their traditions, values, and
customs on the people with whom they traded. They frequently acted without regard to the long-term welfare
of others as their principal concern was short-term economic gain. Since many nations that traded with Europe
placed high value on their historical customs, some natives became deeply disconcerted by the changes that
occurred as a result of European power. These factors, coupled with perennial domestic political instability,
caused numerous countries to grow increasingly resistant to European influence.

One potent example of this ideological shift can be seen in the actions of the Tokugawa government of Japan.
In its Seclusion Edict of 1636, the government attempted to extricate cultural interactions with Europe from
the intimate fabric of Japanese society. The Edict attempted to accomplish this by focusing on three areas.
First, it sought to curb cultural exchange by eliminating people bringing European ideas into Japan. The Edict
stated, "Japanese ships shall by no means be sent abroad…. All Japanese residing abroad shall be put to death
when they return home." Second, the Edict focused on limiting trade. Articles 11 through 17 of the Edict
imposed stringent regulations on trade and commerce. Third, the government banned Christianity, which it
saw as an import from Europe that challenged the long-established and well-enshrined religious traditions of
Japan. The government went to considerable lengths to protect its culture. Article eight of the Edict stated,
"Even ships shall not be left untouched in the matter of exterminating Christians."

124
With the example of Japan and the examples of other countries that chose a different response to European
influence, it is perhaps not too far of a stretch to conclude that Japan made the right decision in pursuing a
path of relative isolationism. As history unfolded during the next 400 years, in general, countries that
embraced European hegemony, whether by choice or by force, tended to suffer from pernicious wealth
inequality, perennial political instability, and protracted underdevelopment.

125
Q1. It can best be inferred from the passage that in 1636, the Japanese government:
A. Saw its citizens living abroad as potential threats
B. Considered all foreign religions a danger
C. Disagreed with the European philosophy that trade brought wealth
D. Foresaw the economic dangers of European trade and imperialism
E. Believed that ideas coming into Japan via foreign interactions provided no positive impact to
Japanese society
Q2. Which of the following best characterizes the most significant motivation for Europe's
behaviour with Japan during the 17th century?
A. Religious zeal
B. Long-term political concerns
C. Short-term economic self-interest
D. Cultural imperialism
E. Territorial aggrandizement
Q3. The author most likely included the quotation from Article Eight of the Edict at the end of
the second paragraph to:
A. Highlight the venomous anger many Japanese leaders felt toward the importation of foreign
religions
B. Emphasize the determination of the Japanese government to protect itself from foreign influences
it saw as damaging
C. Illustrate how pervasive foreign religious influence had become in Japanese society
D. Emphasize that European economic influence offered no justification for the Edict and the
government relied instead on foreign religious influence to justify the Edict
E. Provide an example of Japan's effort to curb cultural and economic exchange
Q4. Based upon the passage, the author would likely agree most strongly with which of the
following statements:
A. European decisions made during the 16th and 17th centuries in dealing with Japan represent an
aberration from the typical pattern of European decisions
B. Japanese rulers who responded with ferocity to European influence bear part of the responsibility
for the caustic European-Japanese relationship that ensued
C. With the hindsight of history, Japan likely made the appropriate decision in extricating itself from
European influence
D. European religious and cultural values conflicted with European economic behaviour toward
Japan
E. The width and breadth of Japan's cultural fabric suffered from its seclusionist policies.

126
UNIT 5
Direct Indirect Speech
What is Direct & Indirect Speech?

Direct Speech: the message of the speaker is conveyed or reported in his own actual words without any
change. It is also called as REPORTED SPEECH.

Indirect Speech: the message of the speaker is conveyed or reported in our own words.

Example on Process of Conversion from Direct to Indirect Speech

a) Direct: Radha said, ―I am very busy now.‖

b) Indirect: Radha said that she was very busy then.

Key Terminology

During the process, you will come across many important terms that you need to know better so that you can
convert any direct speech into indirect speech easily and without any hassle. Consider the following
sentences:

● Direct Speech: She says, ―I am a little bit nervous.‖

● Indirect Speech: She says that she is a little bit nervous.

Now consider the different grammatical aspects of both.

● Reporting Speech: The first part in the direct speech is called reporting speech.

● Reporting Verb: The verb of the reporting speech is called the reporting verb.

Direct speech Indirect speech

Direct speech reports or quotes the exact Indirect speech is also known as reported
words spoken. speech.

Its purpose is to report exactly that was said Its purpose is to report on what someone said

127
by someone by using quotation marks. in past.

It is accompanied by a reporting verb, signal It uses reporting verbs such as ―say,‖ ―tell,‖
phrase, or quotative frame. ―said,‖ ―talk,‖ and ―speak.‖

You will keep the verb tense the same in the Normally, you will change the verb tense
quotations. when using indirect speech

✔ All inverted commas or quotation marks are omitted and the sentence ends with a full stop.

✔ Conjunction ‗that‘ is added before the indirect statement.

✔ The Pronoun is changed in Person.

✔ Present Tense is changed to Past.

✔ The adverb ‗now‘ is changed to ‗then‘.

Tips on Direct and Indirect Speech

Tip 1: Conversion Rules as per the Reporting Verb

When the reporting or principal verb is in the Past Tense, all Present tenses of the direct are changed into
the corresponding Past Tenses.

a) Direct: He said, ―I am unwell.‖

b) Indirect: He said (that) he was unwell.

If the reporting verb is in the Present or Future Tense, the tenses of the Direct Speech do not change.

a) Direct: He says/will say, ―I am unwell.‖

b) Indirect: He says/will say he is unwell.

The Tense in Indirect Speech is NOT CHANGED if the words within the quotation marks talk of a
universal truth or habitual action.

a) Direct: They said, ―We cannot live without water.‖

b) Indirect: They said that we cannot live without water.

Tip 2: Conversion Rules of Present Tense in Direct Speech

128
Simple Present Changes to Simple Past

a) Direct: "I am happy", she said.

b) Indirect: She said that she was happy.

Present Continuous Changes to Past Continuous

a) Direct: "I am reading a book", he explained.

b) Indirect: He explained that he was reading a book.

Present Perfect Changes to Past Perfect

a) Direct: She said, "He has finished his food―.

b) Indirect: She said that he had finished his food.

Present Perfect Continuous Changes to Past Perfect Continuous

a) Direct: "I have been to Gujarat", he told me.

b) Indirect: He told me that he had been to Gujarat.

Tip 3: Conversion Rules of Past & Future Tense

Simple Past Changes to Past Perfect

a) Direct: He said, ―Ira arrived on Monday."

b) Indirect: He said that Ira had arrived on Monday.

Past Continuous Changes to Past Perfect Continuous

a) Direct: "We were living in Goa", they told me.

b) Indirect: They told me that they had been living in Goa.

Past Perfect remains Past Perfect

a) Direct: David said, ―I have not met her‖.


b) Indirect: David said that he had not met her.

Past Perfect Continuous remains Past Perfect Continuous

a) Direct: Jill said, ―I had been cooking‖.

b) Indirect: Jill said that she had been cooking.

Future Simple Changes to Present Conditional

a) Direct: He said, "I will be in Kolkata tomorrow."

129
b) Indirect: He said that he would be in Kolkata the next day.

Future Continuous Changes to Conditional Continuous

a) Direct: She said, "I'll be using the car next Friday.‖

b) Indirect: She said that she would be using the car next Friday.

Future Perfect, will have changes to would have

a) Direct: He said, ―I will have cleaned the room‖.


b) Indirect: He said that he would have cleaned the room.

Future Perfect Continuous remains Future Perfect Continuous (Will changes to would)

a) Direct: She said, "We will have been watching a movie.‖

b) Indirect: She said that they would have been watching a movie.

Tip 4: Changes in Modals

CAN changes into COULD

a) Direct: He said, "I can swim."

b) Indirect: He said that he could swim.

MAY changes into MIGHT

a) Direct: He said, "I may buy a house.‖

b) Indirect: He said that he might buy a house.

MUST changes into HAD TO

a) Direct: He said, "I must work hard.‖

b) Indirect: He said that he had to work hard.

Modals that DO NOT Change: Would, Could, Might, Should, Ought to.

a) Direct: He said, "I should face the challenge.‖

b) Indirect: He said that he should face the challenge.

Tip 5: Conversion of Interrogative

Reporting Verb like ‗said/ said to‘ changes to asked, enquired or demanded

a) Direct: He said to me, ―What are you doing?‖

130
b) Indirect: He asked me what I was doing.

If sentence begins with auxiliary verb, the joining clause should be if or whether.

a) Direct: He said, ―Will you come for the meeting?‖

b) Indirect: He asked them whether they would come for the meeting.

If sentence begins with ‗wh‘ questions, then no conjunction is used as the "question-word" itself act as
joining clause.

a) Direct: ―Where do you live?‖ asked the girl.

b) Indirect: The girl enquired where I lived.

Interrogative sentence in indirect speech ends with a FULL STOP and not a QUESTION MARK.

Tip 6: Command, Request, Exclamation, Wish

Commands and Requests

Indirect Speech is introduced by some verbs like ordered, requested, advised and suggested. Forbid(s)/
forbade is used for the negative sentences. The imperative mood is changed into the Infinitive.

a) Direct: Rafique said to Ahmed, ―Go away.‖

b) Indirect: Rafique ordered Ahmed to go away.

c) Direct: He said to her, ―Please wait.‖

d) Indirect: He requested her to wait.

Exclamations and Wishes

Indirect Speech is introduced by some words like grief, sorrow, happiness, applaud. Exclamatory sentence
changes into assertive sentence and Interjections are removed.

a) Direct: He said, ―Alas! I am undone.‖

b) Indirect: He exclaimed sadly that he was broke.

Tip 7: Change of Pronouns

The first person of the reported speech changes according to the subject of reporting speech.

a) Direct: She said, ―I am in ninth class.‖

b) Indirect: She says that she was in ninth class.

The second person of reported speech changes according to the object of reporting speech.

131
a) Direct: He says to them, "You have completed your job.‖

b) Indirect: He tells them that they have completed their job.

The third person of the reported speech doesn't change.

a) Direct: He says, "She is in tenth class.‖

b) Indirect: He says that she is in tenth class.

Tip 8: Change of Place and Time

Words expressing nearness in time or place in Direct Speech are generally changed into words expressing
distance in Indirect Speech.

Now -- then

Here -- there

Ago -- before

Thus -- so

Today -- that day

Tomorrow -- the next day

This -- that

Yesterday -- the day before

These -- those

Hither-- thither

Come -- go

Hence -- thence

Next week/month -- following week/month

a) Direct: She said, ―My father came yesterday.‖

b) Indirect: She said that her father had come the day before.

c) Direct: She says/will say, ―My father came yesterday.‖

Indirect: She says/will say that her father had come yesterday. (Here the reporting verb ‗says‘ is in the
present tense OR ‗will say‘ is in future tense; hence the time expression ‗yesterday‘ won‘t change.)

Tip 9: Punctuation

132
The words that are spoken should be enclosed in quotes and begin with a capital letter

Example: He said, ―You are right.‖

Comma, full stop, question mark, or exclamation mark must be present at the end of reported sentences
and are placed inside the closing inverted comma or commas.

Example: He asked, ―Can I come with you?‖

If direct speech comes after the information about who is speaking, comma is used to introduce the piece
of speech, placed before the first inverted comma.

Example: She shouted, ―Stop talking!‖

Example: ―Thinking back,‖ she said, ―he didn't expect to win.‖ (Comma is used to separate the two
reported speech and no capital letter to begin the second sentence).

Tip 10: Conversion of Indirect to Direct Speech

1. Use the reporting verb, "say" or "said to" in its correct tense.

2. Remove the conjunctions "that, to, if or whether etc" wherever necessary.

3. Insert quotation marks, question mark, exclamation and full stop, as per the mood of the sentence.

4. Write the first word of the statement with capital letter.

5. Change the past tense into present tense wherever the reporting verb is in the past tense.

Example

a) Indirect: He asked whether he is coming.

b) Direct: He said to him, ―Are you coming?‖

PRACTICE

Change the following into indirect speech

Simple sentence

1. The girl said, ‗It gives me great pleasure to be here this evening.‘

2. The man said, ‗I must go as soon as possible.‘

3. The teacher says, ‗If you work hard, you will pass.‘

4. He said, ‗I have won.‘

133
5. She said, ―I work in an office‖.

Interrogative

1. She said to me, ‗Are you coming with us?‘

2. She said to me, ‗Are you unwell?‘

3. She said to him, ‗Am I to wait for you till eternity?‘

4. I said to him, ‗Were you present at the meeting yesterday?‘

5. The woman asked the stranger, ‗Should I help you?‘

Imperative sentence

1. The said to me, ―Please help me‖


2. She said to him, ―You should find a job‖
3. They said to him, ―Do not go there‖
4. He said, ―Open the door‖
5. The teacher said to student, ―Do not make a noise‖

Exclamatory sentences

1. He said, ―Hurrah! I got a reward‖


2. She said, ―Alas! I failed in exam‖
3. John said, ―Wow! What a nice car it is‖
4. She said, ―Hurrah! I am selected for the job‖
5. He said, ―Oh no! I missed the bus‖

Choose the correct option

Q1. I said to him, ―Will you go to Delhi?‖

(1) I asked him will he go to Delhi.

(2) I said to him would he go to Delhi.

(3) I asked him if he would go to Delhi.

(4) I said to him would you go to Delhi.

Q2. He said, ‗I have read this novel.‘

(1) He said that he has read this novel.

(2) He said that he had read that novel.

(3) He said that he read that novel.

(4) He said that he had read this novel.

134
Q3. Tania said to her friend, ‘Can you lend me an umbrella?‘

(1) Please give me an umbrella Tania requested her friend.

(2) Will you lend me your umbrella; Tania asked her friend.

(3) Tania requested her friend to lend her an umbrella

(4) Tania asked her friend to give her an umbrella.

Q4. Sita said, ‗I may go there.‘

(1) Sita says that she may go there.

(2) Sita says that she Is going there.

(3) Sita said that she will go there.

(4) Sita said that she might go there.

Q5. My friend requested me to bring him a sandwich.

(1) He said, ‗My friend, please bring me a sandwich.‘

(2) My friend said, ‗Will you bring me a sandwich.‘

(3) ‗Please bring me a sandwich‘, said my friend.

(4) ‗Please bring my friend a sandwich‘, said he.

Q6. Kiran asked me, "Did you see the Cricket match on television last night?"

A. Kiran asked me whether I saw the Cricket match on television the earlier night.

B. Kiran asked me if I had seen the Cricket match on television the earlier night.

C. Kiran asked me did I see the Cricket match on television the last night.

D. Kiran asked me if I had seen the Cricket match on television the last night.

Q7. David said to Anna, "Mona will leave for her native place tomorrow."

A. David told Anna that Mona will leave for her native place tomorrow.

B. David told Anna that Mona left for her native place the next day.

C. David told Anna that Mona would be leaving for her native place tomorrow.

D. David told Anna that Mona would leave for her native place the next day.

135
Q8. The tailor said to him, "Will you have the suit ready by tomorrow evening?"

A. The tailor asked him that he will have the suit ready by the next evening.

B. The tailor asked him that he would had the suit ready by the next evening.

C. The tailor asked him if he would have the suit ready by the next evening.

D. The tailor asked him if he would like to have the suit ready by the next evening.

Q9. He said to interviewer, "Could you please repeat the question?"

A. He requested the interviewer if he could please repeat the question.

B. He requested the interviewer to please repeat the question.

C. He requested the interviewer to repeat the question.

D. He requested the interviewer if he could repeat the question.

Q10. He said, "Be quite and listen to my words.‖

A. He urged them to be quite and listen to his words.

B. He urged them and said be quite and listen to his words.

C. He urged they should be quite and listen to his words.

D. He said you should be quite and listen to his words.

Q11. He said to me, I have often told you not to play with fire.

A. He said that he has often been telling me not to play with fire.

B. He told me that he had often told me not to play with fire.

C. He reminded me that he often said to me not to play with fire.

D. He said to me that he often told me not to play with fire.

Q12. Pawan said to me, "If I hear any news, I'll phone you."

A. Pawan told me that if he heard any news, he will phone me.

B. Pawan told me that if he will hear my news, he will phone me.

C. Pawan told me that if he had heard any news, he would phone me.

D. Pawan told me that if he heard any news, he would phone me.

Q13. Mohan said, "We shall go to see the Taj in the moonlit night":

136
A. Mohan said that we shall go to see the Taj in the moonlit night.

B. Mohan told that we shall go to see the Taj in moonlit night.

C. Mohan said that we should go to see the Taj in the moonlit night.

D. Mohan said that they would go to see the Taj in moonlit night.

Q14. The teacher said to Ram, "Congratulations! Wish you success in life."

A. The teacher congratulated Ram and said wish you success in life.

B. The teacher wished congratulations and success in life to Ram.

C. The teacher wished congratulations to Ram and wished him success in life.

D. The teacher congratulated Ram and wished him success in life.

Q15.The poor examinee said, "Oh God, take pity on me."

A. The poor examinee prayed God to take pity on him.

B. The poor examinee, involving God, implored him to take pity on him.

C. The poor examinee exclaimed that God take pity on him.

D. The poor examinee asked God to take pity on him.

Q16. "Where will you be tomorrow," I said, "In case I have to ring you?"

A. I asked where you will be the next day in case, I will ring him.

B. I asked where he would be the next day in case, I had to ring him.

C. I said to him where he will be in case i have to ring him.

D. I enquired about his where about the next day in case I would have to ring up.

Q17. The father warned his son that he should be beware of Angad.

A. The father warned his son, "Beware of Angad!"

B. The father warned his son, "Watch that chap!"

C. The father warned his son, "Be careful about Angad."

D. The father warned his son, "Don't fall into the trap."

Q18. Farhan asked Geeta, "Could you lend me a hundred rupees until tomorrow?"

A. Farhan asked Geeta if she could lend him a hundred rupees until tomorrow.

137
B. Farhan asked Geeta if she could lend him a hundred rupees until the next day.

C. Farhan asked Geeta if she could lend me a hundred rupees until the next day.

D. Farhan asked if Geeta could lend me a hundred rupees until the next day.

Q19. "What about going for a swim," he said, "It's quite fine now."

A. He asked me what about going for a swim as it was quite fine then.

B. He proposed going for swim as it was quite fine.

C. He suggested going for a swim as it was quite fine.

D. He advised me to go for a swim as it was quite fine.

Q20. "You can't bathe in this sea," he said to me, "it's very rough."

A. He told me that I can't bathe in this sea because it's very rough.

B. He told me that you couldn't bathe in the sea if it was very rough.

C. He told me that I couldn't bathe in that sea as it was very rough.

D. He told me that you can't bathe in this sea since it was very rough.

COVER LETTER
The purpose of a cover letter, which should always accompany your resume, is to introduce yourself and
compel an employer to learn more about you through your resume.

138
PERSONALIZE

The statement ―To Whom it May Concern‖ is concerning! Always address your letter to a specific person. If
you do not know who should receive your letter, research the organization‘s Web site or call the main
number and ask for the appropriate person‘s name and title.

RESEARCH

In addition to determining the recipient‘s name and title, research the organization so that you can convey an
appreciation for its mission and an understanding of the duties and qualifications of the position for which
you are applying.

CONVINCE

Articulate how your skills and experiences uniquely qualify you for the position and demonstrate why you
are a good fit for the organization. Be clear and concise and limit your cover letter to one page. Additionally,
maintain a professional tone while providing insight into your personality so that your letter leaves an
employer with a sense of wanting to learn more about you.

PROOFREAD

A poorly written or error-laden letter is a surefire way to end your candidacy. If you want an employer to
spend additional time reviewing your resume, you must make time to proofread your letter and ensure that
your grammar and spelling are perfect.

COVER LETTER FORMAT

Date

Ms./Mr./Dr. recipient‘s first and last name

Title

Organization name

Street address

City, state, zip code

Dear Ms./Mr./Dr. Last Name

Paragraph 1: State why you are writing, how you learned of the organization or position, and basic
information about yourself. If you are writing at the suggestion of someone who knows the recipient, say so.

139
Paragraph 2: Demonstrate your knowledge and interest in the organization, and use specific examples to
show how your background and skills qualify you for the position.

Paragraph 3: This paragraph is optional and is recommended for elaborating on particularly relevant or
impressive details included in your resume.

Paragraph 4: Indicate that your resume and other supporting materials are enclosed. Reiterate your interest
in the position and your desire to meet for an interview. State your plans to follow up via e-mail or phone
within a certain period and invite the employer to contact you to request additional information. Lastly,
thank the employer for his/her consideration.

Sincerely

Your name

SAMPLE 1: Computer science cover letter

October 4, 2018

Ms. Lori Hopkins

VP of Information Technology

ABC Corporation

245 Maple St. Sometown, NJ 55555

Dear Ms. Hopkins

I found your posting on Shine.com for a network administrator and I wanted to apply immediately. ABC
Corporation‘s reputation for responsive service and proactive support differentiates your company from the
rest, and I would like to be part of your dynamic team.

Since graduating from Lovely Professional University with a BS in computer science, I have worked for
three years as a network administrator for DEF Company. In this role, I am valued as a solutions-focused
manager of networks, IT systems, user support functions and technology projects. I have experience in all
areas covered in your job ad, including:

● Network and Server Design and Administration

● LAN, WAN, VPN, SAN and VoIP Infrastructure

● Network Analysis and Optimization

● System Security, Disaster Recovery and Business Continuity Plans

● Cloud Computing and Data Storage

● Enterprise System and Software Implementations

140
Known as a keen technical trouble-shooter, I am dedicated to providing cost-effective and expedient
solutions optimizing network stability and security; improving performance of systems and equipment; and
resolving issues such as network crashes, system slowdowns and virus outbreaks.

At DEF Company, I provide world-class service technical support in an enterprise environment, and I am
confident I would quickly become a productive network administrator at ABC Corporation. If you agree,
please call me at (555) 555-5555 to set up an interview.

I look forward to your response.

Sincerely

Barbara Holt

SAMPLE 2: Computer science cover letter

5TH May,2019

Ms. Elisa Thompson

XYZ Company

Hartford, CT 56789

Dear Ms. Thompson

When I discovered the software engineering internship with XYZ Company on shine.com, I was excited by
the opportunity to develop test cases, open bug reports, and triage test case failures. As a sophomore
majoring in Computer Science at University College, I enjoy developing and improving testing
technologies. My coursework, employment, and campus involvement have prepared me well for this
position.

● Coursework. After completing courses in software development methods, programming languages,


network systems, and embedded system design, my Major GPA is 3.8.

● IT Help Desk employment. As a student employee with IT services, I troubleshoot a wide variety of
computing issues and communicate technical information to non-technical customers.

● Campus involvement. As a member of the Association for Women in Computing, I attend Tech Talks
and conferences to expand my industry knowledge and network with full-time professionals.

I have experience with Java, Python, JavaScript, and SQL. This knowledge will be an asset to this position. I
would be thrilled to learn more alongside your cross-functional team of engineers and developers.

My enclosed resume expands on my coursework, employment, and campus involvement. As I prepare for a
career in computer science, I am dedicated to gaining practical experience to complement my coursework.
Thank you for your time and consideration. I look forward to hearing from you soon.

Sincerely

141
Peyton Java

SAMPLE 3: ELECTRICAL ENGINEERING

[Application Date]

Mr. Justin Shaw

Robotics Startup

Chicago, IL 56789

Dear Mr. Shaw

When I discovered the electrical engineering internship with Robotics Startup on internships.com, I was
excited by the opportunity to develop electrical and electronic design for autonomous robots and mobility
platforms. As a junior majoring in Electrical Engineering at University College, I enjoy designing circuits
and experimenting with new technologies. My academics, campus involvement, and volunteer work have
prepared me well for this position.

● Academics. After completing courses in Computer Aided Measurement and Controls, Circuit Analysis,
Calculus, Physics, and Electronics, my Major GPA is 3.9.

● Campus involvement. As a member of the Robotics Club, I have gained experience with hardware, data
processing, sensors, microcontroller programming, and communication protocols.

● Volunteer work. For the past two years, I have mentored a high school robotics team. In preparation for
a Robotics Competition, the students build and program an industrial robot.

I have experience creating circuit schematics and PCB layouts. These skills will be an asset to this position,
and I would be thrilled to learn alongside your interdisciplinary team of innovators.

My enclosed resume expands on my academics, campus involvement, and volunteer work. As I prepare for
a career in electrical engineering, I am dedicated to gaining practical experience to complement my
coursework. Thank you for your time and consideration. I look forward to hearing from you soon.

Sincerely

Adrian Ware

SAMPLE 4: COVER LETTER FOR MECHANICAL ENGINEERING BRANCH

[Date]

[Name of HR Manager]

[Position Title]

142
[Company Name]

[Company Address]

[City, State, Zip]

Dear Mr./Ms. [...]

Please accept this letter and the accompanying resume as an expression of my interest in a position with
your organization.

As my resume indicates, in June 2006, I expect to receive a Bachelor of Science degree in mechanical
engineering from California Polytechnic State University. I would then like to begin employment with an
organization in which my effective performance will be met with recognition and growth.

I have acquired a sound overall knowledge of leading-edge engineering principles, tools, and practices, with
emphasis on designing, building and testing of mechanical systems. I am proficient in the use of various
automated solutions including current releases of AutoCAD and SolidWorks.

I have applied classroom learning to innovative and successful projects in which I served as sole or principal
designer. Additionally, I have been a responsible leader in a family-owned small business.

My personal attributes include leadership and sound judgment as well as creativity, analytical and
troubleshooting skills. I interact productively with people from diverse backgrounds. I have a career history
of achieving employment goals. I have a history of quality work carried to timely completion.

I am certain I could make significant contributions to your organization, and I would welcome the
opportunity to meet with you to discuss how my education and abilities might best be employed by your
organization.

Thank you for your consideration. I look forward to your response.

Sincerely

Mark Gunlogson

SAMPLE LETTER 5: MECHANICAL ENGINEERING

5TH May,2019

Ms. Elisa Thompson

XYZ Company

Hartford, CT 56789

Dear Ms. Thompson

143
Please accept this letter and the accompanying resume as an expression of my interest for the Mechanical
Engineer position offered by your organization.

I Graduated as a master's in mechanical engineering from university of Alabama in Huntsville. As a


mechanical engineer, I have a strong background in the basic principles of engineering, methods and
practices in mechanical system design, testing and troubleshooting mechanical equipment. In addition, I am
proficient in using FEA software‘s & advanced CAD designing tools.

My professional work experience as graduate trainee engineer at manufacturing industry and my years of
academic background have given me not only a strong technical foundation but also domain knowledge and
exposure to Work environment. My key strengths include effective communication, strong troubleshooting
skills, Quick problem-solving ability and passionate to learn and adopt new technologies and skills.

As my years of education and experience make me feel confident that I can excel and live up to the
expectations in performing the duties assigned. My enclosed resume provides more details on my
qualifications, skills and work experience.

I would welcome the opportunity to meet with you to discuss how my education and experience might best
be employed to your organization. Please feel free to contact me if you have any further questions. I will be
available as soon as possible to take up the responsibilities if you find me as an potential fit for this position.

Sincerely

Mark Federick

SAMPLE 6: CIVIL ENGINEERING

January 8, 2019

Ms. Diane Smith

HR Manager

ABC Company

55 Circle Point

Sometown, AZ 55555

Dear Ms. Smith

Your civil engineer trainee position is an exciting opportunity. ABC Company‘s stellar reputation for
providing quality service and project support to your clients is well-known, and I am motivated to join your
team.

Highlights of my credentials:

144
● BS in civil engineering and Engineer in Training (EIT) certification from NCEES.

● Two years of experience as a civil engineer trainee and intern for premier design firms.

● Hands-on experience providing project design and site civil engineering leadership for water,
wastewater, roadway and other municipal infrastructure projects in Sometown, AZ.

● Delivery of on-schedule, on-budget and high-quality completion of infrastructure projects with


varying degrees of complexity and budgets ranging from $500K to $6.8M.

● Proficient in preparing drawings and technical specs using AutoCAD Civil 3D and MicroStation.

In addition, I have worked closely with licensed PEs to save our clients and employers tens of thousands of
dollars while maintaining compliance with regulatory requirements. I have provided creative, cost-effective
design solutions to issues including erosion, corrosive groundwater, insufficient water inflow/drainage, soil
instability, steep-slope grading and other engineering challenges.

If my credentials and strong understanding of municipal water and wastewater infrastructure systems are
well matched to your needs, please contact me at (555) 555-5555 or email [email protected] to
schedule an interview. Thanks very much.

Sincerely

Lena Marks, EIT

SAMPLE: CIVIL ENGINEERING

Date: 22 October 2018

Mr. Philip Smith

Hiring Manager

Richmond Engineering Services

London

E12 5DD

Dear Mr. Smith

I wish to apply for the post of Civil Engineer as advertised on the Dayjob.com web site. The position seems
an ideal opportunity for me as it matches my experience, knowledge and career aspirations.

As you can see from my CV, I have amassed over 7 years of significant, progressive experience in civil
engineering projects within the construction and gas industries. During my career I have participated in more
than 20 civil engineering projects, all of them were successful.

145
My progressive engineering experience has provided me with the opportunity to develop strong client
relation building skills and an ability to lead multi-disciplinary teams. I am experienced at applying the
principles of civil engineering when planning, designing, and overseeing the construction and maintenance
of structures and facilities.

I enclose my resume for your review and look forward to meeting with you in the near future. Should you
have any questions or require clarification on any information please contact me at the above telephone
number.

I would welcome the opportunity to talk to you and I hope that you will invite me for an interview. I thank
you for your time and I look forward to hearing from you.

Yours sincerely

Richard Daniels

444 Hanley Road

Birmingham

B18 6NF

Mobile: 0121 638 0026

Email: [email protected]

COVER LETTER FOR SCIENCE GRADUATES

[date]

Mr. Rover Tulin

HR Manager

Nestle Ghana Limited

Tema, Greater Accra

Dear Mr.Tulin

Please accept my application for the position of Food Scientist as advertised in the Daily Graphic.

I am a first-degree holder in Agricultural Science from the Kwame Nkrumah University of Science and
Technology. During my National Service at Accra Brewery, I worked as an Assistant Food Production
Scientist. My job experience is in the area of molecular manufacturing technology and the use of
nanotechnology in food sensor development.

I possess top research skills with the ability to liaise effectively in a team.

146
My attention to detail and excellent problem-solving skills ensures that every project I work on is done
accurately and to the highest possible standard. My verbal communication skills and positive attitude make
me an effective communicator with all clients I work with.

I look forward to the opportunity of discussing my application with you further and how I can significantly
contribute to the ongoing success of your company.

Yours sincerely

Michael Lamondo

SAMPLE LETTER FOR SCIENCE:

Date

Ms. Joy Franklin

California State University

1442 Dark Hollow Road

Camden, NJ 8102

Dear Ms. Joy Franklin,

Highly motivated individual seeking employment in the agriculture industry where I can put my skills and
training to good use by helping to solve some of the problems the world faces today.

I have a real desire to learn more about wildlife and a concern for the health and well-being of the different
species located all over the globe. The animals and plants play a major role in our future. It is my hope to
help find ways for humans and wildlife to exist together without major consequences by reducing the impact
humans have on the animals as our habitats and technology continues to grow.

I earned a bachelor‘s degree in wildlife management from California State University and completed an
internship to gain hands-on experience in this profession. My training focused on conservation,
environmental management and wildlife ecology and management.

I can conduct research to identify problems associated with the fisheries and wildlife and to find suitable
solutions for these problems. My abilities include excellent verbal and written skills that help me to convey
the need for concern and to explain why changes are needed to protect our future. I also can compile reports
that show the results of my research, along with possible solutions that can help solve some of these
problems.

Other skills that make me suitable for a position in this industry includes having lots of patience along with
excellent time management and multitasking skills and a real desire to make a difference.

147
Please call (111)-494-6970 if you have a position available.

Respectfully

Signature

William Mitchell

Practice:

QUESTIONS FOR COMPUTER SCIENCE


1. Write a cover letter for applying to a job in Tata Consultancy service, considering yourself as a
fresher.
2. Write a cover letter for the post of Software Engineer, giving the details of the experience and
achievements.
3. Write a cover letter for the post of Data Analyst in Adobe in Bangalore. Inform Ms. Lori
Hopkins, VP of Information Technology, Adobe,245 Maple St., Bangalore.
4. Write a cover letter for the post of Hardware Engineer, giving the details of the experience and
achievements.
5. Write a cover letter to the Robotics Pvt Ltd, to apply for the internship in the Artificial intelligence
and neurological networks. Also, mention your achievements and subjects of interest.

QUESTIONS FOR MECHANICAL ENGINEERING

1. Write a cover letter for the post of Design Engineer in XYZ Pvt Ltd, Bangalore. Write the letter to
Mr.PR Rattan who is the Manager of the company. Also, mention about the software you have
worked on for designing.
2. Write a cover letter to the Robotics Pvt Ltd, to apply for the internship in the field of Industrial
engineering and testing. Also, mention about the project which you were handling for testing the
machines and various projects.
3. Write a cover letter to Ms. Elisa Thompson from XYZ Company at Hartford, regarding the
vacancy for Automobile engineer. Also, mention about your research work, achievements and
projects done till yet.
COVER LETTER FOR CIVIL ENGINEERING

148
1. Write a cover letter to Ms. Anna Thompson from XYZ Company at Hartford, regarding the
vacancy for civil engineer. Also, mention about your research work, achievements and projects
done till yet.

2. Write a cover letter for the post of Design Engineer in XYZ Pvt Ltd, Bangalore. Write the letter to
Mr.PR Rattan who is the Manager of the company. Also, mention about the software and tools you
have knowledge and use for designing.

3. Write a cover letter for applying to a job in Anand Construction company, considering yourself as a
fresher.

COVER LETTER FOR ELECTRONICS ENGINEERING.

1. Write a cover letter for the post of Junior embedded software engineer in the ABC Pvt ltd, Pune.
Write about the experience, achievements you have attained and also about the projects done till
yet.

2. Write a cover letter to the Robotics Pvt Ltd, to apply for the internship in the Artificial intelligence
and neurological networks. Also, mention your achievements and subjects of interest.

3. Write a cover letter to HBL Power Systems Limited, for the post of electronic engineer. Write this
letter as a fresher and do mention the projects you did in your bachelor degree.

COVER LETTER FOR AGRICULTURE


1. Write a cover letter for the post of Farm manager in the Imagine People Solutions Pvt Ltd in
Gwalior. Mention your experience, projects and internship.

2. Write a cover letter for the post of Soil Scientist in the Reeracoen India Pvt Ltd, Bangalore. Write
this letter as a fresher and do mention about the projects and research papers.

3. Write to the Dharitri Rural AGRO-TECH Pvt Ltd for the post of Plant breeder. Write your previous
experience, reason behind leaving the previous industry.

149
UNIT 6
Critical Reasoning
Critical thinking is the ability to think clearly and rationally about what to do or what to believe. It
includes the ability to engage in reflective and independent thinking. Someone with critical thinking skills is
able to do the following:

● understand the logical connections between ideas

● identify, construct and evaluate arguments

● detect inconsistencies and common mistakes in reasoning

● solve problems systematically

● identify the relevance and importance of ideas

● reflect on the justification of one's own beliefs and values

Example of a Critical Reasoning Text

A CEO of a major company noted a serious decline in worker productivity during the previous five years.
According to a report done by an outside consultant, productivity dropped by 35% by the end of that period.

150
The CEO has therefore initiated a plan to boost productivity by giving employees shares of the company as
part of their pay package.

Conclusion

Most problems have a central idea or thesis. This is almost always located in the sentence at the beginning of
the text or in the sentence at the very end. In this case, it is at the end of the passage: The CEO has therefore
initiated a plan to boost productivity by giving employees shares of the company as part of their pay
package. Words like therefore, thus, hence, and so usually tell us that this is the conclusion.

Premise

Premises are the facts or evidence that support or lead to the conclusion. Unlike assumptions, they are
explicit. Here is an example from the text: A CEO of a major company noted a serious decline in worker
productivity during the previous five years. This premise helps the author lead to the conclusion or main
idea of the text.

Assumption

Assumptions are the facts that support the conclusion, like the premise does, but unlike the conclusion and
premises they are not stated in the text, they are implicit. Here is what would be an example of an
assumption for this Critical Reasoning problem: Owning something or part of something obliges you work
harder to make it succeed.

Supporting Information

Like a premise, this is stated, and explicit information embedded in the text, but unlike a premise, it does not
support the conclusion. At best, it supports a premise or provides further detail or information regarding a
premise. From the text: According to a report done by an outside consultant, productivity dropped by 35%
by the end of that period.

Arguments in real life can take several forms, but arguments on Critical Reasoning questions are relatively
formulaic. The typical argument has three parts:

1) Premise: the starting point of deductions; often, agreement to this is assumed.

2) Conclusion: what the author wants you to believe by the end of the argument

3) Assumption: the unstated link between premise and conclusion. Although unstated, the assumption is the
nerve center of the argument, the linchpin holding the whole thing together.

Premise (s) + Assumption (s) = Conclusion

Argument: Every hockey fan I know is nice. I do not know Judy, but since she is wearing a hockey jersey,
she must be nice.

Premise: Every hockey fan I know is nice. I don‘t know Judy. Judy is wearing a hockey jersey.

Assumption: Since Judy is wearing hockey jersey, she is a fan.

151
Conclusion: Judy is nice

Types of Critical Reasoning Questions

Critical Reasoning Questions can be categorized into five major types.

1. Weaken the Argument

2. Strengthen the Argument

3. Find the Assumption

4. Find the Conclusion / Draw Inference

5. Paradox Questions

Premise vs. Conclusion

A premise includes the reasons and evidence behind a conclusion. A conclusion is the statement that the
premise supports and is a way of promoting a certain belief or point of view. To help us better identify
the premise and conclusion of an argument, we can look at indicator words.

Consider the following argument: Since carrots are full of vitamins, it follows that your body will
benefit if you eat them.

In this argument, how do we know which part is considered the premise and which part is the
conclusion? The premise here is the fact that carrots are full of vitamins. The conclusion is that your
body will benefit from you eating carrots.

This statement about carrots includes indicator words. Indicator words aid you when you are trying to
identify an argument and its parts. The phrase Since carrots are full of vitamins uses the indicator word
'since' which is often associated with premises. The last part of the sentence uses the phrase, 'it follows
that' to show that it is a conclusion.

Examples of words or phrases that are typically included in premises:

Because since given that seeing that as shown by assuming that


considering that for the reason that

Examples of words or phrases that are typically included in conclusions:

Therefore thus it follows that which proves/implies that which means that as a result
so we may conclude

If the passage contains no indicator words, try these two strategies:

a. Ask yourself, "What claim is the writer or speaker trying to prove?" That claim will be the
conclusion.

152
b. Try putting the word "therefore" before each of the statements in turn. The statement it fits best will
be the conclusion.

Practice: Identify premise and conclusion in the following arguments and label them as (p) and (c)
respectively.

1. No brass instruments use reeds, and flutes don't use reeds, so flutes must be brass instruments.

2. Betty will be angry unless someone else brings the music stands. But if Ann doesn't bring them, no one
will. So, either Ann will bring the music stands or Betty will be angry.

3. If we don't consolidate city and county school systems, the city school system will continue to
deteriorate, producing many young adults who are not equipped to find work that will keep them out of
poverty. We must not allow this disastrous social situation to occur, so we must consolidate city and county
schools.

4. Many herbs are known to have medicinal properties. So, it is not a waste of taxpayer dollars to finance
trials of herbal treatments that appear implausible. Open-mindedness is a friend of scientific investigation,
and since herbal treatments are widely used, it is important to determine whether they work and whether
they have any harmful effects.

5. A steady movement of people from the city to suburban and rural areas has decreased the city's
population, increased the percentage of its population that are poor, and increased the percentage of its
population that are African American and Hispanic. If the tax base continues to shrink, then the resulting
poor support for education and services, combined with racial and ethnic polarization, will lead to increasing
tension between city and suburban populations. We must stop this trend. The only way to stop it is by
consolidating city and county governments, so that there is a single tax base in support of a unified, high-
quality system of education and a uniform level of municipal services.

Answers:

1. [P1] No brass instruments use reeds, and [P2] flutes don't use reeds, so [C1] flutes must be brass
instruments.

2. [P1] Betty will be angry unless someone else brings the music stands. But [P2] if Ann doesn't bring them,
no one will. [C1] So either Ann will bring the music stands or Betty will be angry.

3. [P1] If we don't consolidate city and county school systems, the city school system will continue to
deteriorate, producing many young adults who are not equipped to find work that will keep them out of
poverty. [P2] We must not allow this disastrous social situation to occur, so [C1] we must consolidate city
and county schools.

4. [P1] Many herbs are known to have medicinal properties. So [C1]it is not a waste of taxpayer dollars to
finance trials of herbal treatments that appear implausible. Open-mindedness is a friend of scientific
investigation, and since [P2] herbal treatments are widely used, [C2] it is important to determine whether
they work and whether they have any harmful effects.

153
5. [P1] A steady movement of people from the city to suburban and rural areas has decreased the city's
population, increased the percentage of its population that are poor, and increased the percentage of its
population that are African American and Hispanic. [P2] If the tax base continues to shrink, then the
resulting poor support for education and services, combined with racial and ethnic polarization, will lead to
increasing tension between city and suburban populations. [P3] We must stop this trend. [P4] The only way
to stop it is by consolidating city and county governments, so that there is a single tax base in support of a
unified, high-quality system of education and a uniform level of municipal services.

Weaken the Argument/Find the Flaw

There are two ways in which you can weaken an argument:

I. An answer that weakens the argument can directly disprove the assumption. Any choice that states that the
assumption is wrong will weaken the argument.

II. If different evidence can strengthen the argument; different evidence can weaken the argument as well.
Any new information given in an answer choice that assumes less likely to be correct will weaken the
argument.

A Weaken the Argument question may be worded as:

⮚ Which of the following, if true, most seriously weakens the argument?


⮚ Which of the following, if true, would cast the most doubt on…?
⮚ The objection implied above…is based on doubts about…?
⮚ Which of the following points to the most serious logical flaw in the author‘s argument?
Read the sample question given below to get more clarity:

The cellular service quality has dropped significantly in India. Ten years ago, there were no instances of call
drop or no connectivity. There is also a decrease in the speed and reliability of service.

All of the following would tend to strengthen the conclusion of the argument above except:

A. The volume of connections handled by the Mobile operators has increased dramatically over the last ten
years.

B. Unprecedented increases in the cost as well as scarce availability of spectrum for mobile services have
put severe pressures on the Mobile companies.

C. Mobile services have diversified from carrying mere voice data to a whole range of internet data such as
downloading, video-calling, data sharing, etc.

D. The opposition to negative externality of cellular radiation has obstructed increase of network services in
response to the increasing subscriber base.

Understand the question carefully. The question is asking you look for an option that will weaken our
conclusion. For that you need to find the conclusion. The conclusion in the given argument is 'The cellular
service quality has dropped significantly in India.'

154
Now if we look at the option A, it is uprightly giving us a reason as why there is a decline in the services. If
the volume of the mobile connection subscriber increases in a dramatic way, and the service provider is not
in a capacity to handle it perfectly, it will surely lead to mismanagement.

And similar is the case with option B, as- if there is scarcity of spectrum it will be difficult to give good
services.

We are now left with option C and D. Option C says that as compared to the older times when the cellular
companies were catering to only one kind of service whereas, now they have diversified their services. This
option is weakening the main argument, which makes it the correct answer option.

Option D is also strengthening the conclusion as it says that due to the negative externality of the radiation,
people are opposing it but they are not opposing the use of mobile phones, which means there may be fewer
mobile towers in comparison to the increasing subscriber base.

Strengthen the Argument

If you are asked to look for an answer choice that strengthens the argument; then find the answer choice that
corroborates that the central assumption is correct.

There are two ways of strengthening an argument:

I. If you come across an answer choice that would be correct according to an assumption question, it would
also be correct for a strengthen question. The correct answer can simply be a paraphrasing of the assumption
itself.

II. If the correct answer can confirm the assumption by mentioning an experiment, survey or any other proof
and helps the assumption appear to be true, then it will support our main argument.

A Strengthen the Argument question may be worded as:

⮚ Which of the following, if true, most strongly supports…?

⮚ Which of the following, if true, would most strengthen…?

⮚ The statements above, if true, best support which of the following assertions?

Read the sample question given below to get more clarity:

Beasley &Halpert Law Firm has instituted an Employee Wellness Program that will provide attorneys and
support staff with free access to Smokers Anonymous programs, diabetes monitoring, and discounted
memberships to a local gym. Similar programs at other firm have been shown to improve workplace
attendance and performance and reduce the employer‘s costs for employee health insurance. Thus, the
Employee Wellness Program will be good for both the employees and the firm.

If true, which of the following would best support the conclusion of the argument above?

155
a) Many employees take advantage of free diabetes monitoring when it is offered by employers.

b) Smokers Anonymous programs are only effective for 20% of those smokers who use them.

c) Discounted memberships at a local gym will make it easier for employees to improve their cardiovascular
health and reduce the incidence of serious illness.

d) Exercising without the help of a personal trainer can often lead to injury due to incorrect use of weight-
training equipment.

e) Beasley &Halpert will give employees taking part in the Smokers Anonymous program one paid hour off
each Friday afternoon to attend the group meetings.

Choice A - The monitoring‘s popularity might indicate that it is beneficial to the employees, but it might not.
This choice doesn‘t clearly demonstrate that the Employee Wellness Program benefits the employees and is
a good example of a wrong answer.

Choice B makes it LESS likely that the programs will benefit either the employees or the firm. This answer
choice may catch your eye if you didn‘t read the question closely enough and are mistakenly looking for a
weakener instead of a strengthener.

Choice C is the correct answer. The argument seems to imply that the increased attendance and performance
and reduced health insurance costs are due to improved employee health, which would naturally benefit the
employees. This choice makes that unstated implication clear and fills the gap in the argument.

Choice D might be tempting, but again, it requires too many assumptions to tie it into the argument as a
strengthener. For this to strengthen the argument, one must assume that employees would still exercise
without the personal training services, and that they would incorrectly use the weight-training equipment.

Choice E might benefit the employees, but it would be a burden to Company X. Therefore, it‘s not the best
choice.

Find the Assumption

Assumption questions will ask you to select the answer choice with the information that must be true (the
‗assumption‘) for the given argument to be accurate. To figure out the answer, you‘ll first need to figure out
the main thrust of the given argument. The correct answer choice, again, will have to be true for that
argument to be logical. Incorrect answer choices will often be possibly true but won‘t be necessary to the
argument‘s validity.

A Find the Assumption question may be worded as:

⮚ Which of the following is an assumption made in drawing the conclusion above?

⮚ Which of the following in an assumption on which the argument relies?

Read the sample question given below to get more clarity:

156
Last year, support for the social and behavioural sciences represented only about three percent of the
government‘s total budget for research funds in the United States. Thus, the particularly sharp reductions
imposed on such programmes this year seem to be dictated not by financial constraints but by social
philosophy.

Which of the following is an assumption on which the conclusion of the above passage is based?

A. Government funding is the primary source for research money in the United States.

B. The social and behavioural sciences are as valuable as physical and biological sciences.

C. Three per cent is an insignificant portion of the government‘s total budget for research funds.

D. The government funds allocated for research in the social and behavioural sciences are not sufficient
for the work that needs to be done.

It‘s a simple assumption question. The conclusion, quite clearly, is that the reduction is dictated not by
financial concerns, but by social philosophy. The reasoning here is that since these fields are a mere three
percent of the total money spent on research, it would be foolish to make drastic reductions purely for
financial reasons. The basis of the reasoning is obviously that three percent of the money spent on research
isn‘t really a significant sum – which is what option C states. C is the correct answer.

While answering a question, you often find that many options seem extremely relevant to the given
situation. Remember then, your job is not to find what is relevant, but to deduce the basis of the argument.

The Negation Test

This test always works, and it always a good way to verify an assumption. Here‘s the rule:

If you negate a statement, and it‘s still possible to imagine that the conclusion is still true even with
this negated statement, then that original statement is not an assumption of the argument.

If you negate a statement, and this negated statement is a devastating objection which shatters the
argument and makes the conclusion untenable, then that original statement is an assumption of the
argument.

Here‘s a super-simple argument, with only three answer choices:

3) Alex likes this movie. Therefore, Betty will like it.

Find the assumption of the argument

A. Both Alex & Betty liked the same movie last year

B. Carla didn‘t like this movie, and last year, she & Betty liked the same movie.

C. Betty likes the movies that Alex likes.

Find the Conclusion/Draw Inference

157
Inference questions ask you to make inferences—draw logical conclusions—based on the evidence in the
given passage. Key words you might see are imply or infer.

An Inference question may be worded as:

⮚ Which of the following can properly be inferred from the statements above?
⮚ Which of the following, if true, best supports as a conclusion??
For inference questions, you must draw conclusions only from the information you‘re directly given in
the passage. Beware of answer choices that contain words like ‗any,‘ ‗best,‘ ‗worst,‘ ‗only,‘ ‗all,‘ or ‗none,‘
as they are often overly general and can‘t be verified by the limited information in the passage. Incorrect
answer choices will often be overly extreme; make leaps in logic that can‘t be verified based on the given
info or contain tangential/unrelated information.

Read the sample question given below to get more clarity:

Randall: Many of the productions of my plays by amateur theater groups are poorly done, and such
interpretations do not provide a true measure of my skills as a dramatist.

Which one of the following can be properly inferred from Randall‘s statement?

(A) Some amateur theater groups‘ productions of Randall‘s plays provide a true measure of his skills as a
dramatist.

(B) All amateur theater group productions of Randall‘s plays that are not poorly done provide a true measure
of his skills as a dramatist.

(C) All of the productions of Randall‘s plays by amateur theater groups that do not provide a true measure of
his skills as a dramatist are poorly done.

(D) If a production of a dramatist‘s play is well done, then it provides a true measure of his or her skills as a
dramatist.

(E) At least some amateur theatrical groups‘ productions of Randall‘s plays fail to provide a true measure of
his skills as a dramatist.

(A) seriously distorts Randall‘s statement. Just because some amateur productions don‘t do him justice
doesn‘t mean that there are other productions that do. If the GMAT tells you that some marbles are red, you
can‘t automatically infer that some are not red.

(B) is another sort of distortion. Randall‘s statement about certain poorly done productions in no way
guarantees anything about productions that aren‘t poorly done.

(C) is far too extreme. Randall does establish a correlation between poor production quality and failure to
provide a true measure of his skills, but that correlation has only been established for a certain set of
productions and can‘t be extended to all productions.

158
(D) attempts to extract a broad principle from Randall‘s statement, but his statement is too particular to
allow this kind of extrapolation.

The answer is (E).

Paradox Questions

Paradox/Discrepancy questions will ask you to choose the answer choice that explains the paradox in the
given argument. A paradox refers to the coexistence of two seemingly contradictory pieces of information.
The correct answer choice will logically explain why those pieces of information are not actually
contradictory. Key words you might see are explain, paradox, or discrepancy.

A Paradox question may be worded as:

⮚ Which of the statements below provides the most likely explanation for the two seemingly
contradictory statements above?

⮚ Which of the following, if true, most helps to resolve the paradox outlined above?

⮚ Which of the following, if true, best explains the reason for the apparent discrepancy described
above?

Read the sample question below to get the clarity:

From 1998 to 2008, the amount of oil exported from the nation of Livonia increased by nearly 20% as the
world‘s demand soared. Yet over the same period, Livonia lost over 8,000 jobs in oil drilling and
refinement, representing a 25% increase in the nation‘s unemployment rate.

Which of the following, if true, would best explain the discrepancy outlined above?

A. Because of a slumping local economy, Livonia also lost 5,000 service jobs and 7,500 manufacturing jobs.
B. Several other countries in the region reported similar percentages of jobs lost in the oil industry over the
same period.

C. Because of Livonia‘s overvalued currency, most of the nation‘s crude oil is now being refined after it has
been exported.

D. Technological advancements in oil drilling techniques have allowed for a greater percentage of the
world‘s oil to be obtained from underneath the ocean floor.

E. Many former oil employees have found more lucrative work in the Livonia‘s burgeoning precious metals
mining industry.

PRACTICE

159
Q1. Thousands of people contract tonsillitis every year, and yet all go on to live normal lives after the
operation. We can conclude, from this observation, that the tonsils have no function in the body.

The argument would be most weakened by which of the following, if it were true?

a) People live normal lives after appendectomies, but the appendix is known to be part of the digestive
system.

b) The tonsils have been shown to have a vital role to play in the physiology of laboratory rabbits and
mice.

c)The human tonsil develops as part of the immune system, a system of vital importance in defense
against disease.

d)Another part of the body can take over the function of the tonsils if they are removed.

e) Tonsillectomies are performed only when the tonsils become seriously infected.

Q2. According to an article in a nutritional magazine, eating beets significantly lowers the risk of cancer.
The article refers to a study that found that people who consumed one or more beets per day were
half as likely to be diagnosed with cancer as people who did not.

Which of the following, if true, most weakens the argument in the magazine article?

a) The study was only conducted in one city.

b) The participants in the study who ate beets were more likely to exercise regularly than those who did
not eat beets.

c) In another experiment, cancer patients who ate one or more beets per day were no more likely to
recover than those who ate no beets.

d) Participants in the study reported consuming no vegetables other than beets.

e) Another study found that people who consumed one tablespoon of fish oil per day were more than
four times less likely to be diagnosed with cancer as those who did not.

Q3. The average life expectancy for the United States population is 73.9 years, but children born in
Hawaii will live an average of 77 years, and those born in Louisiana, 71.7 years. If a newlywed
couple from Louisiana were to begin their family in Hawaii, therefore, their children would be
expected to live longer than would be the case if the family remained in Louisiana.

Which of the following statements, if true, would most significantly strengthen the conclusion drawn in
the passage?

A. As population density increases in Hawaii, life expectancy figures for that state are likely to be revised
downward.

B. Environmental factors tending to favor longevity are abundant in Hawaii and less numerous in
Louisiana.

160
C. Twenty-five percent of all Louisianans who move to Hawaii live longer than 77 years.

D. Over the last decade, average life expectancy has risen at a higher rate for Louisianans than for
Hawaiians.

E. Studies show that the average life expectancy for Hawaiians who move permanently to Louisiana is
roughly equal to that of Hawaiians who remain in Hawaii.

Q4. Even though most universities retain the royalties from faculty members' inventions, the faculty
members retain the royalties from books and articles they write. Therefore, faculty members should
retain the royalties from the educational computer software they develop.

The conclusion above would be more reasonably drawn if which of the following were inserted into the
argument as an additional premise?

A. Royalties from inventions are higher than royalties from educational software programs.

B. Faculty members are more likely to produce educational software programs than inventions.

C. Inventions bring more prestige to universities that do books and articles.

D. In the experience of most universities, educational software programs are more marketable than books
and articles.

E. In terms of the criteria used to award royalties, educational software programs are more nearly
comparable to books and articles than to inventions

Q5. A medical degree is necessary for appointment to the hospital's board of directors. Further, no one
having more than a five-percent equity stake in a pharmaceutical company can be appointed to the
board of directors. Consequently, Dell, a practicing physician with a PhD in bioethics, cannot be
appointed the hospital's treasurer, since he owns fifteen percent of PillCo, a pharmaceutical
company.

The argument‘s conclusion follows logically if which one of the following is assumed?

a) A PhD is not necessary for appointment to the position of treasurer.

b) PillCo is one of the hospital's pharmaceutical vendors.

c) If Dell sold his stake in PillCo, he would be appointed treasurer.

d) Only those eligible for appointment to the hospital's board of directors can be appointed as the
hospital's treasurer.

e) Anyone with a medical degree who does not hold more than a five-percent stake in any
pharmaceutical company is eligible for appointment to the hospital's board of directors.

161
Q6. To prevent some conflicts of interest, Congress could prohibit high-level government officials from
accepting positions as lobbyists for three years after such officials leave government service. One
such official concluded, however, that such a prohibition would be unfortunate because it would
prevent high-level government officials from earning a livelihood for three years.

The official's conclusion logically depends on which of the following assumptions?

A. Laws should not restrict the behavior of former government officials.

B. Lobbyists are typically people who have previously been high-level government officials.

C. Low-level government officials do not often become lobbyists when they leave government service.

D. High-level government officials who leave government service can earn a livelihood only as lobbyists.

E. High-level government officials who leave government service are currently permitted to act as
lobbyists for only three years.

Q7. Opponents of laws that require automobile drivers and passengers to wear seat belts argue that in a
free society people have the right to take risks as long as the people do not harm other as a result of
taking the risks. As a result, they conclude that it should be each person's decision whether or not to
wear a seat belt.

Which of the following, if true, most seriously weakens the conclusion drawn above?

A. Many new cars are built with seat belts that automatically fasten when someone sits in the front seat.

B. Automobile insurance rates for all automobile owners are higher because of the need to pay for the
increased injuries or deaths of people not wearing seat belts.

C. Passengers in airplanes are required to wear seat belts during takeoffs and landings.

D. The rate of automobile fatalities in states that do not have mandatory seat belt laws is greater than the
rate of fatalities in states that do have such laws.

E. In automobile accidents, a greater number of passengers who do not wear seat belts are injured than
are passengers who do wear seat belts.

Q8. Increase in the level of high-density lipoprotein (HDL) in the human bloodstream lower bloodstream-
cholesterol levels by increasing the body's capacity to rid itself of excess cholesterol. Levels of HDL
in the bloodstream of some individuals are significantly increased by a program of regular exercise
and weight reduction. Which of the following can be correctly inferred from the statements above?

A. Individuals who are underweight do not run any risk of developing high levels of cholesterol in the
bloodstream.

B. Individuals who do not exercise regularly have a high risk of developing high levels of cholesterol in
the bloodstream late in life.

162
C. Exercise and weight reduction are the most effective methods of lowering bloodstream cholesterol
levels in humans.

D. A program of regular exercise and weight reduction lowers cholesterol levels in the bloodstream of
some individuals.

Q9. People are always less happy to accept scientific data they feel contradicts their preconceived
beliefs. No surprise here; no human likes to be wrong. But science isn't supposed to care about
preconceived notions. Science, at least good science, tells us about the world as it is, and not as
some wish it to be. Sometimes what science finds is consistent with a particular religion's wishes. But
usually, it is not.

What can be inferred about good science? Select from the given options.

a) A good science is well received by the educated people.

b) A good science is based on concrete results obtained through testing the hypothesis.

c) A good science and religion are same.

d) A good science will always prove the general populace wrong.

Q10. Between 1960 and 1970, ivory poachers in the African nation of Zinbaku killed over 6,500
elephants. During that period, the total elephant population in Zinbaku fell from about 35,000 to just
under 30,000. In 1970, new anti-poaching measures were implemented in Zinbaku, and between
1970 and 1980 over 800 poachers were arrested and expelled from the country. Nevertheless, by
1980, the elephant population in Zinbaku had fallen to about 21,000.

Which of the following, if true, would best help to explain the apparent paradox presented above?

a) The poachers arrested in Zinbaku between 1970 and 1980 were rarely sentenced to long prison terms

b) Because of highly publicized campaigns against the slaughter of elephants, demand for ivory fell
between 1970 and 1980

c) The elephant population in neighbouring Mombasa rose slightly between 1970 and 1980

d) In Zinbaku, between 1970 and 1980, thousands of acres of forest, the elephant's natural habitat, were
cleared for farming

Q11. A survey conducted recently in the city indicated that most college welfare-aid applicants
understate the number of luxury items - such as cars and TVs – that their family owned, in an effort
to maximize the amount of aid they can claim from the city. Paradoxically, the same study also found
that many applicants claimed that they had running water and a gas connection even when they did
not.

Which of the following best explains the apparent paradox?

163
a) The city does not pay welfare unless the applicants have at least some things working for them.

b) Claiming that they do not have a car, or a TV ensures that the city looks at the applicant more
favorably.

c) While the applicants may be willing to accept that they don't have certain things, they felt
embarrassed having to accept that they don't have most things.

d) Historically, at least 30℅ of the claims have had people understating what they have while only 22℅
overstated what they had.

e) The people who understated what they had were not the same people who overstated what they had.

Q12. At an enormous research cost, a leading chemical company has developed a manufacturing
process for converting wood fibers into a plastic. According to the company, this new plastic can be
used for, among other things, the hulls of small sailboats. But what does the company think sailboat
hulls used to be made of? Surely the mania for high technology can scarcely go further than this.

Which of the following, if true, would most seriously weaken the author‘s conclusion?

a) The wood used in producing the plastic is itself in increasingly short supply.

b) The plastic produced by the process is considerably lighter, stronger, and more watertight than wood.

c) The cost of the manufacturing process of the plastic increases the cost of producing a sailboat hull by
10 to 15 percent.

d) Much of the cost of the research that developed the new process will be written off for tax purposes
by the chemical company.

Q13. A law requiring companies to offer employees unpaid time off to care for their children will harm
the economic competitiveness of our nation‘s businesses. Companies must be free to set their own
employment policies without mandated parental-leave regulations.

Which of the following, if true, would most seriously weaken the conclusion of the argument
above?

a) Many businesses in this country already offer employees some form of parental leave.

b) A parental-leave law will serve to strengthen the family as a social institution in this country.

c) In most polls, a majority of citizens say they favor passage of a parental-leave law.

d) Some of the countries with the most economically competitive businesses have strong parental-leave
regulations.

Q14.Which of the following best completes the passage below?

People buy prestige when they buy a premium product. They want to be associated with something
special. Mass-marketing techniques and price-reduction strategies should not be used because____.

164
A. affluent purchasers currently represent a shrinking portion of the population of all purchasers

B. continued sales depend directly on the maintenance of an aura of exclusivity

C. purchasers of premium products are concerned with the quality as well as with the price of the
products

D. expansion of the market niche to include a broader spectrum of consumers will increase profits.

Q 15. Which of the following best completes the passage below? Established companies concentrate on
defending what they already have. Consequently, they tend not to be innovative themselves and tend
to underestimate the effects of the innovations of others.

The clearest example of this defensive strategy is the fact that___.

A. ballpoint pens and soft-tip markers have eliminated the traditional market for fountain pens, clearing
the way for the marketing of fountain pens as luxury or prestige items

B. a highly successful automobile was introduced by the same company that had earlier introduced a
model that had been a dismal failure

C. a once-successful manufacturer of slide rules reacted to the introduction of electronic calculators by


trying to make better slide rules

D. one of the first models of modern accounting machines, designed for use in the banking industry, was
purchased by a public library as well as by banks

16. A researcher discovered that people who have low levels of immune-system activity tend to score
much lower on tests of mental health than do people with normal or high immune-system activity.
The researcher concluded from this experiment that the immune system protects against mental
illness as well as against physical disease.

The researcher's conclusion depends on which of the following assumptions?

A. High immune-system activity protects against mental illness better than normal immune-system
activity does.

B. Mental illness is similar to physical disease in its effects on body systems.

C. People with high immune-system activity cannot develop mental illness.

D. Mental illness does not cause people's immune-system activity to decrease.

E. Psychological treatment of mental illness is not as effective as is medical treatment.

Q17. Traditionally, decision-making by managers that is reasoned step-by-step has been considered
preferable to intuitive decision-making. However, a recent study found that top managers used

165
intuition significantly more than did most middle-or lower-level managers. This confirms the
alternative view that intuition is more effective than careful, methodical reasoning.

The conclusion above is based on which of the following assumptions?

(A) Methodical, step-by-step reasoning is inappropriate for making many real-life management
decisions.

(B) Top managers can use either intuitive reasoning or methodical, step-by-step reasoning in making
decisions.

(C) The decisions made by middle-and lower-level managers can be made as easily by using methodical
reasoning as by using intuitive reasoning.

(D) Top managers use intuitive reasoning in making most of their decisions.

(E) Top managers are more effective at decision-making than middle-or lower-level managers

Q18. The ancient city of Cephesa was not buried by an eruption of Mt. Amnos in A.D. 310, as some
believe. The eruption in the year 310 damaged the city, but it did not destroy it. Cephesa survived for
another century before it finally met its destruction in another eruption around A.D. 415.

Which of the following, if true, would most strengthen the author‘s claim that the city of Cephesa
was not buried by the eruption of Mt. Amnos in A.D. 310?

a) The city of Cephesa is mentioned in a historical work known to have been written in A.D. 400.

b) Coins bearing the image of an emperor who lived there around A.D. 410 have been discovered in the
ruins of Cephesa, which were preserved by the cinders and ashes that buried the city.

c) Geological evidence shows that the eruption of Mt. Amnos in A.D. 415 deposited a 10-foot-thick
layer of lava on the city of Cephesa.

d) A historical work written in A.D. 430 refers to the eruption of Mt. Amnos in A.D. 415.

Q19. Reva: Using extraneous incentives to get teenagers to change their attitude toward school and
schoolwork won‘t work. Take the program in West Virginia, for instance, where they tried to reduce
their dropout rate by revoking the driving licenses of kids who left school. The program failed
miserably.

Anne: It‘s true that the West Virginia program failed, but many schools have devised incentive programs
that have been very successful in improving attendance and reducing discipline problems.

According to Anne, the weak point in Reva‘s claim is that it

a) fails to consider the possibility that the majority of potential dropouts in West Virginia do not have
driving licenses

b) doesn‘t provide any exact figures for the dropout rate in West Virginia before and during the
program

166
c) ignores a substantial body of evidence showing that parents and employers have been using extrinsic
incentives with positive results for years

d) is based on a single example, the incentive program in West Virginia, which may not be typical

Q20. Correctly measuring the productivity of service workers is complex. Consider, for example, postal
workers: they are often said to be more productive if more letters are delivered per postal worker.
But is this true? what if more letters are lost or delayed per worker at the same time that more are
delivered?

The objection implied above to the productivity measure described is based on doubts about the truth of
which of the following statements?

(A) Postal workers are representative of service workers in general.

(B) The delivery of letters is the primary activity of the postal service.

(C) Productivity should be ascribed to categories of workers, not to individuals.

(D) The quality of services rendered can appropriately be ignored in computing productivity.

(E) The number of letters delivered is relevant to measuring the productivity of postal workers.

167
168

You might also like